Anda di halaman 1dari 46

1.

An AIDS patient, who is being treated with multiple drugs, develops breast hypertrophy, central adiposity, hyperlipidemia, insulin resistance
and nephrolithiasis. If these changes are related to his drug treatment, this drug belongs to which group of anti-retroviral drugs?
a) Nucleoside Reverse Transcriptase Inhibitors (NRTIS)
b) Non-nucleoside Reverse Transcriptase Inhibitors (NNRTIS)
c) Fusion Inhibitors
d) Protease Inhibitors
e) Integrase Inhibitors
1. A 35 year old HIV positive male patient comes to the OPD with complaint of anorexia, nausea and vomiting and abdominal pain. His
abdomen is tender in the epigastric area. Laboratory results reveal a raised serum amylase activity and a preliminary diagnosis is made of acute
pancreatitis. Which of the following anti-retroviral drugs has the patient most likely been taking?

a) Saquinavir
b) Zidovudine
c) Didanosine
d) Efavirenz
e) Enfuvirtide

2. A 40 year old HIV positive patient is receiving HAART regimen (Highly active anti-retroviral therapy). Four weeks after initiating therapy, he
comes to the emergency department complaining of severe pain in the flank, nausea and frequent urination. Which one of the following drugs is
most likely the cause of his symptoms?
a) Zidovudine
b) Indinavir
c) Efavirenz
d) Nevirapine
e) Nelfinavir

3. A 30 year old man is recently diagnosed with HIV and therapy is started. After the first week of therapy, the patient complains of headaches,
irritability, and nightmares. Which one of the following anti-retroviral drugs is most likely to be causing these symptoms?
a) Efavirenz
b) Indinavir
c) Lamivudine
d) Nevirapine
e) Stavudine

4. A 35 year old woman is diagnosed with chronic hepatitis B infection and therapy is initiated. Just after a few hours she comes to the
emergency department complaining of fever, chills and muscle aches. Which one of the following drugs most likely caused these symptoms?
a) Lamivudine
b) Adefovir
c) Entecavir
d) Interferon alfa
e) Ribavirin

6. A 60 year old man with known history of Parkinson’s disease is to receive prophylaxis against Influenza A virus. He is given a drug that is
useful against Parkinson’s disease as well for prophylaxis against influenza. The drug with which of the following mechanisms of action is most
likely to have been given?

a) It prevents entry and penetration of the virus


b) It prevents uncoating of the virus
c) It prevents replication of the virus
d) It prevents assembly of newly synthesized virus particles
e) It prevents release of the newly synthesized virus
Q7. Antimicrobial agent of choice for the outpatient treatment of infections due to animal bite wound be:
a) Cefuroxime sodium
b) Amoxicillin/clavulanate potassium
c) Penicillin V
d) Ampicillin sodium/Salbactam sodium
e) Ticarcillin disodium/clavulanate potassium
Q8. An antibiotic is distributed in total body water and has an elimination half life of 45 minutes. Which one of the following statements is
consistant with that observation?
a) The drug is stored in high concentration in fat
b) The rapid rate of disappearance rules out metabolism of the drug by the liver
c) The drug is actively transported into tubular urine
d) The drug cannot be bound to plasma proteins
e) The drug is eliminated only by glomerular filtration
Q9. Which one of the following is indicated for treatment of chlamydial urethritis during pregnancy?
a) Amoxicillin (Amoxil)
b) Penicillin V
c) Erythromycin base
d) Doxycycline (Monodox)
e) Tetracycline
Q10. A 25-year-old male has a dental infection associated with facial swelling and lymphadenopathy. Which one of the following is the most
appropriate antibiotic?
a) Cephalexin
b) Tetracycline
c) Penicillin (note: watch an animation on penicillin mechanism of action)
d) Erythromycin
e) Gentamicin
Q11. A 16-year-old sexually active nulliparous white female complains of pelvic pain and vaginal discharge. On examination she is found to have
a temperature of 39.8° C (102.0° F), pain with movement of the cervix, and tenderness and a mass in the right adnexa. Which one of the
following treatment would be appropriate?
a) Outpatient treatment with penicillin G procaine ,intramuscularly; probenecid orally; plus doxycycline (Vibramycin) orally for 14 days and
reexamination in 3 days
b) Outpatient treatment with ceftriaxone intramuscularly; probenecid orally; plus doxycycline twice a day for 14 days and reexamination in 1
week
c) Outpatient treatment with cefoxitin intramuscularly; plus doxycycline twice a day for 14 days and reexamination in 10 days
d) Hospitalization for treatment with cefoxitin intravenously and doxycycline orally or intravenously, then doxycycline orally twice a day to
complete 14 days of treatment
e) Erythromycin 2 grams orally as a single dose
Q12. A drug that may cause nephrotoxicity, is:
a) Penicillin G
b) Erythrocin
c) Gentamycin
d) Cefuroxime
e) Penicillin V
Q13. Which one of the following is a common early side effect of Penicillin?
a) Constipation
b) Loss of appetite
c) Orthostatic hypotension
d) Atrioventricular block
e) Skin rash
Q14. Which of the following might be seen in patient of TB, who has regularly been injecting intramuscular injection of streptomycin:
a) Depression
b) Sialorrhea
c) Increased serum alanine aminotransferase
d) Priapism
e) Deafness
Q15. Significant negative interaction has been known to occur between the following drugs if given concurrently:
a) Penicillin G & Penicillin V
b) Penicillin & tetracycline
c) Penicillin & Gentamycin
d) Penicillin & Clavulanic acid
e) Ticarcillin & Clavulanic acid
Q16. Which of the following is NOT a recognized complication of ampicillin toxicity:
a) Hemolytic anemia
b) Diarrhea
c) Nausea
d) Overgrowth of gram-positive organisms
e) Ventricular fibrillation

Q17. Doctor is wrong when he suggests his patient to get drug:


a) Streptomycin by IV route
b) Gentamycin by IV route
c) Neomycin by oral route
d) Penicillin V by oral route
e) Cefepime by IV route

Q18. Telithromycin:
a) Is structurally related to tetracycline
b) Is structurally related to beta lactam antibiotics
c) Is structurally related to aminoglycosides (note: see an animation on aminoglycosides mechanism of action)
d) Binds more tightly to ribosomes and so it is a poor substrate for bacterial efflux pumps that mediateantibiotic resistance
e) Is used for UTI
Q19. Doxycycline is used:
a) for Pneumonia as drug of 1st choice
b) for the prevention of TB
c) for the prevention of malaria
d) for the prevention of leprosy
e) for the treatment of sexually transmitted diseases

Q20. Clindamycin:
a) is chemically related to macrolides
b) binds to 30S ribosomal subunit
c) is not recommended for oral route
d) cross-resistance may be observed between macrolides and Clindamycin
e) is excreted in breast milk

Q21. Which of the following statement is true:


a) nafcillin is beta-lactamase resistant antibiotic
b) cefazolin is second generation cephalosporin
c) tazobactam is used in gonorrhea mainly
d) penicillin G is contraindicated in infections caused by spirochetes
e) thrombocytosis is main side effect of Linezolid
Q22. All of the following statement is true, except:
a) chloramphenicol is broad spectrum protein synthesis inhibitor
b) linezolid and streptogramins are narrow spectrum protein synthesis inhibitors
c) mechanism of resistance to chloramphenicol is plasmid mediated and occurs through the formation of acetyltransferases that inactivate the
drug
d) tetracyclines are narrow spectrum protein synthesis inhibitors
e) susceptible organisms accumulate tetracyclines intracellularly via energy dependant transport systems in their cell membranes
Q23. in a patient with culture-positive enterococcal endocarditis who has failed to respond to vancomycinbecause of resistance, the treatment
most likely to be effective is:
a) clarithromicin
b) erythromycin
c) linezolid
d) minocycline
e) Ticarcillin
Q24. A 24 year old male patient is suffering from peptic ulcer. He was advised anti-peptic ulcer drugs including doxycycline. Which one of the
following statements about doxycycline is false?
a) it is bacteriostatic
b) it is excreted mainly in the feces
c) it is used in Lyme disease
d) it has a long elimination half-life
e) it is more active than tetracycline against H. Pylori

Q25. A 12 years old female patient came to pediatrician suffering from headache, high grade fever, moderate chest pain, joint pain and drowsy.
When doctor got history, he was told to take diazepam 2.5 mg by patient 15 hours ago. Actually it was already a diagnosed case of pneumonia
and she was on drug treatment including streptogramin. Concerning streptogramins, which one of the following statements is false:
a) they are active against methicillin-resistant staphylococci
b) they may cause a syndrome of arthralgia and myalgia
c) they are used in the management of infections caused by vancomycin-resistant enterococci
d) they are associated with post antibiotic effect
e) they induce formation of hepatic drug-metabolizing enzymes
Q26. Telithromycin:
a) is cell wall synthesis inhibitor
b) is a ketolide structurally related to macrolides
c) it binds very loosely to ribosomes so it is good substrate for bacterial efflux pumps that mediate resistance
d) is used in Lyme disease mainly
e) is used in pneumonia as drug of 1st choice

Q27. Which statement is true about tetracyclines?


a) resistance mechanisms include decreased activity of the uptake systems and, most importantly, the development of mechanisms like efflux
pumps for active extrusion of tetracyclines
b) is contraindicated in gram-positive bacterial infections
c) is absolutely contraindicated in gram-negative bacterial infections
d) mechanism of action of tetracycline is same as Clindamycin
e) resistant organisms are killed when tetracycline is used in large doses

Q28. A 33 years old male patient has been diagnosed mycoplsma pneumoniae infection. He was treated by one of the broad spectrum
antibacterial drug group tetracycline 2 gram TID for 21 days. What may be most appropriate toxicity?
a) anemia
b) hepatic necrosis
c) thrombocytosis
d) alopecia
e) sleeplessness

Q29. an elderly debilitated patient has a fever believed to be due to an infection. He has extensive skin lesions, scrapings of which reveal the
presence of large numbers of gram-positive cocci. The most appropriate drug to use for treatment of this patient is:
a) amoxicillin
b) salbactam
c) cefoxitin
d) nafcillin
e) penicillin G

Q30. A 21 year old man was seen in a clinic with a complaint of dysuria and urethral discharge of yellow pus. He has a painless clean-base ulcer
on the penis and nontender enlargement of the regional lymph nodes. Gram stain of the urethral exudates showed gram negative diplococci
within polymorphonucleocytes. The patient informed the clinic staff that he was unemployed and had not eaten a meal for two days. The most
appropriate treatment of gonorrhea in this patient is:
a) amoxicillin orally for 7 days
b) vancomycin intramuscularly as a single dose
c) ceftriaxone intramuscularly as a single dose
d) tetracycline orally for 7 days
e) procaine penicillin G intramuscularly as a single dose plus one gram of probenecid
Q31. Eighty years old male patient having 73 Kg body weight was suffering from fever, headache and lumbar pain since last 7 days. He was
examined in clinic. Gram stain of the smear of CSF revealed gram positive rods resembling diphtheroids. The antibiotic regimen for empiric
treatment would include:
a) Erythromycin
b) Ticarcillin
c) Cefotetan
d) Cefazolin
e) Ampicillin

Q32. A 25 year old male presents to the medical OPD with complaint of cough and low grade fever since the last 3 months. Laboratory tests
show sputum positive for acid fast bacteria. Which of the following combination of drugs is most likely to be administered at the start to this
patient?
a) Streptomycin, Isoniazid, Rifampicin and Pyrazinamide
b) PAS, Pyrazinamide and Rifampicin
c) Pyrazinamide, Ethambutol and Isoniazid
d) Streptomycin and Rifampicin
e) Streptomycin alone

Q33. A 25 year old male has been given treatment for pulmonary tuberculosis for 2 months as initial phase. Now he comes to the OPD for
medication for the continuation phase. Which of the following drugs will he most likely be given for the continuation phase?
a) Pyrazinamide and Isoniazid
b) Ethambutol and Streptomycin
c) Isoniazid and Rifampicin
d) Rifampicin and Streptomycin
e) Ethambutol and Pyrazinamide

Q34. A young man 26 years old is started treated for pulmonary tuberculosis. He comes to the medical OPD after 1 week with complaint of
orange red colored urine and sweat and tears. Which of the following drugs is most likely causing these symptoms?
a) Streptomycin
b) Rifampicin
c) Ethambutol
d) Pyrazinamide
e) Thiacetazone

Q35. A 30 year old male with known TB infection and taking medication since 1 month comes to the Eye OPD with complaint of visual
disturbances including decreased visibility and slight color blindness. If these symptoms are due to his drug treatment, which one of the
following drugs is most likely the cause of his symptoms?
a) Streptomycin
b) Ciprofloxacin
c) Ethambutol
d) Rifampicin
e) Isoniazid

Q36. A 35 year old male recently diagnosed with tuberculosis and is started on medication. After a few days he comes to the emergency
department a few hours after taking a meal at McDonald’s with severe pain in the big toe. Blood tests show a very high uric acid level. Which of
the following drugs most likely caused these symptoms?
a) Streptomycin
b) Rifampicin
c) Isoniazid
d) Amikacin
e) Pyrazinamide

Q37. A 40 year old male taking medication for tuberculosis since the last 2 months comes to the ENT OPD with complaint of hearing problems
and ringing in the years. The drug with which of the following mechanisms of action is most likely causing these symptoms?
a) Binds to 30 S Ribosome subunit & inhibits initiation complex
b) Inhibits DNA Dependent RNA Polymerase
c) Inhibits synthesis of mycolic acid
d) Inhibits synthesis of arabinoglycan subunits
e) Inhibits DNA Gyrase

Q38. A 35 year old female taking medication for tuberculosis and oral contraceptives comes to the Gynaecology OPD with complaint of
amenorrhea. Laboratory test shows a positive pregnancy test. Which one of the following drugs most likely caused her to become pregnant?
a) Rifampicin
b) Pyrazinamide
c) Streptomycin
d) Ethambutol
e) Isoniazid

Q39. A 45 year old male with diagnosed chronic liver disease gets infected with the tuberculosis. Which one of the following drugs would be
most safe for this patient for the treatment of tuberculosis?
a) Isoniazid
b) Rifampin
c) Pyrazinamide
d) Ethambutol
e) Ethionamide

Q40. A 48 year old male is diagnosed with tuberculosis and has been treated for 2 months with isoniazid and rifampicin with good response. But
the patient develops numbness and paresthesias in the extremities. What will you most likely do next?
a) Add pyridoxine
b) Stop isoniazid
c) Add vitamin E
d) Replace rifampin with ethambutol
e) Replace isoniazid with streptomycin
Q41. A 30 year old male presents with history of fever occurring in a step-ladder fashion for the last 10 days. He also feels abdominal pain after
taking meals. Lab tests show decreased TLC count and a positive Widal test. The drug which is most likely to be given acts by which one of the
following mechanisms?
a) Ciprofloxacin
b) Ampicillin
c) Cotrimoxazole
d) Cefixime
e) Azithromycin

Q42. A 35 year old pregnant woman presents to the gynecology department with complaint of pain with rising fever since the last 5 days. Lab
tests show gram negative bacilli and widal test comes out positive. Which one of the following drugs will most likely be administered?
a) Ciprofloxacin
b) Levofloxacin
c) Ofloxacin
d) Norfloxacin
e) Ampicillin

Q43. A 27 year old male football player gets injured in a match. He comes to the emergency department with pain and swelling near the ankle.
On examination, there is small wound with a tender swelling near the ankle joint. Further tests reveal tendon rupture with tendonitis setting in.
Which one of the following antibiotics, if required, will most probably not be given to this patient?
a) Ceftriaxone
b) Clarithromycin
c) Aminoglycoside
d) Ciprofloxacin
e) Amoxicillin

Q44. A 40 year old male comes to the medical OPD with symptoms of cough & fever for the last few days. Sputum culture shows presence
of gram positive cocci. Which one of the following Fluoroquinolones is most likely to be administered to this patient?
a) Ciprofloxacin
b) Norfloxacin
c) Ofloxacin
d) Gatifloxacin
e) Levofloxacin

Q45. A 50 year old patient comes to the emergency department, a day after he has been administered an antibiotic for an on-going infection,
with complaint of palpitations. ECG reveals prolonged QT interval. Which one of the following Fluoroquinolones most likely caused this adverse
effect?
a) Ciprofloxacin
b) Norfloxacin
c) Levofloxacin
d) Ofloxacin
e) Gatifloxacin

Q46. Disinfectants are:


a) Strong chemical agents that inhibit or kill micro-organisms
b) Agents with sufficient toxicity for host cells
c) They kill both vegetative cells and spores
d) A process intended to kill or remove all types of micro-organisms including spores and include viruses
e) A process that kills non-sporulating microorganisms by hot water or steam at 65-100 C

Q47. A patient with watery stools is diagnosed as suffering from amebic dysentery. He is given a drug that causes a metalic taste in the mouth
which drug may be given.
a) Iodoquinol
b) Diloxanide furate
c) Metronidazole
d) Pentamiline
e) Emetine.

Q48. The reason for giving Metronidazole for oropharyngeal infection is due to its good activity against:
a)Gram positive cocci
b)Gram Negative cocci
c)Gram positive bacilli
d)Gram negative bacilli
e)Anaerobes like B. fragilis

Q49. Patient comes to emergency with the complaints of marked visual and auditory abnormities, vomiting diarrhea abdominal pain & skin
rashes. From the history of the patient it was revealed that he took some drug for the treatment of fever. What could be probable diagnosis?
a)Cinchonism
b)Acute gastro enteritis
c)Cholera
d)Typhoid fever
e)Acid peptic disease

Q50. A 37 year old male is having repeated episodes of fever with chills with blood smear positive for malaria since last one year. He was given
chloroquine each time & the fever subsided. Which of the following drugs would you add with chloroquine this time?
a)Artemether
b)Mefloquine
c)Malarone
d)Primaquine
e)Halofantrine

1 – Introduction to Pharmacology: Basic Principles


1) Which of the following is NOT part of the etymology of the word pharmacology?
a) Medicine
b) Drug
c) Herb
d) Poison
e) Study

2.1) Which of the following describes an agonist?


a) Any substance that brings about a change in biologic function through its
chemical action
b) A specific regulatory molecule in the biologic system where a drug interacts
c) A drug that binds to a receptor and stimulates cellular activity
d) A drug that binds to a receptor and inhibits or opposes cellular activity
e) A drug directed at parasites infecting the patient
2.2) Xenobiotics are considered:
a) Endogenous
b) Exogenous
c) Inorganic poisons
d) Toxins
e) Ligands
2.3) Which of the following would be a toxin (poison of biological origin)?
a) Pb
b) As
c) Hg
d) Atropine
2.4) The vast majority of drugs have molecular weights (MW) between 100 and 1,000.
Large drugs, such as alteplase (t-PA), must be administered:
a) Into the compartment where they have their effect
b) Orally so they do not absorb too quickly
c) Rectally to prevent irritation to the stomach lining and vessels
d) Via the intraosseous (IO) route
e) Titrated with buffering agents to prevents cell lysis
2.5) Which of the following occurs with drugs that are extremely small, such as Lithium?
a) Receptor mediated endocytosis
b) Minor drug movement within the body
c) Vasodilation when injected intravenously (IV)
d) Specific receptor binding
e) Nonspecific binding
2.6) Drugs fit receptors using the lock and key model. Covalent bonds are the ____ and
the ____ specific.
a) Strongest; Most
b) Strongest; Least
c) Weakest; Most
d) Weakest; Least
2.7) Warfarin (Coumadin) is given as a racemic mixture with the S enantiomer being four
times more active than the R enantiomer. If the mixture of Warfarin given is 50% S and
50% R, what is the potency compared with a 100% R enantiomer solution?
a) 4 * R + 1 * S = 1
b) 4 * R + 1 * S = 1.5
c) 4 * R + 1 * S = 2
d) 4 * R + 1 * S = 2.5
e) 4 * R + 1 * S = 4
2.8) What determines the degree of movement of a drug between body compartments?
a) Partition constant
b) Degree of ionization
c) pH
d) Size
e) All of the above
3.1) Which of the following is NOT a protein target for drug binding?
a) Side of action (transport)
b) Enzymes
c) Carrier molecules
d) Receptors
e) Ion channels
3.2) Which of the following is an example of a drug acting directly through receptors?
a) Protamine binds stoichiometrically to heparin anticoagulants
b) Adrenergic beta blockers for thyroid hormone-induced tachycardia
c) Epinephrine for increasing heart rate and blood pressure
d) Cancer chemotherapeutic agents
e) Mannitol for subarachnoid hemmorhage
4.1) What is added with drug subclassification, such as an antitubercular drug versus an
antibacterial drug?
a) Cost
b) Size
c) Ionization
d) Precision
e) Speed
4.2) What type of drug is propranolol (Inderal)?
a) Anticonvulsive
b) Antihypertensive
c) Antinauseant
d) Antihistamine
e) Antipyretic
5.1) Which of the following is considered the brand name?
a) Propranolol
b) Inderal
c) Adrenergic ß-blocker
d) “off label” use
e) Blocks ß-receptors in heart myocardium
5.2) Which of the following is considered the class?
a) Propranolol
b) Inderal
c) Adrenergic ß-blocker
d) “off label” use
e) Blocks ß-receptors in heart myocardium
5.3) Which of the following cases would be contraindicated for propranolol (Inderal)?
a) Hypertension
b) Essential tremor
c) Angina
d) Tachycardia
e) Asthma
5.4) Which of the following adverse effects (side-effects) is NOT commonly seen with
cholinergic antagonists?
a) Blurred vision
b) Confusion
c) Miosis
d) Constipation
e) Urinary retention
6.1) The drug chloramphenicol (Chloromycetin) is risky for which of the following?
a) Neonates
b) Geriatric patients
c) Adult males
d) Obese patients
e) Congestive heart failure patients
6.2) How does the glomerular filtration rate (GFR) change after the age of 40?
a) Increase 1% each year
b) Increases 2% each year
c) Decreases 1% each year
d) Decreases 2% each year
e) Does not depend on age
6.3) A decrease in renal and liver function, as seen in the elderly, would prolong drug
half-life, ____ plasma protein binding, and ____ volume of distribution.
a) Increase; Increase
b) Decrease; Decrease
c) Increase; Decrease
d) Decrease; Increase
6.4) When prescribing isoniazid (Rimifon), pharmacogenetics must be considered as
>90% of Asians and certain other groups are ____ acetylators, and thus have a ____
blood concentration of a given dose and a decreased risk of toxicity.
a) Slow; Increased
b) Slow; Decreased
c) Fast; Increased
d) Fast; Decrease
6.5) Which of the following are the two modifying factors that contribute to why women
have higher blood peak concentrations of alcohol than men when consuming equivalent
amounts?
a) Lower blood volume & increased hormones
b) Lower fat content & more gastric alcohol dehydrogenase (ADH)
c) Higher fat content & more gastric alcohol dehydrogenase (ADH)
d) Lower fat content & less gastric alcohol dehydrogenase (ADH)
e) Higher fat content & less gastric alcohol dehydrogenase (ADH)

2 – Pharmacokinetic Principles: Drug Movement


1) Pharmacokinetics is the effect of the ____ and pharmacodynamics is the effect of the ____.
a) Drug on a drug; Body on the drug
b) Body on the drug; Drug on a drug
c) Drug on the body; Body on the drug
d) Body on the drug; Drug on the body
e) Drug on a drug; Drug on a drug

2.1) Which of the following is NOT an action of the body on a drug?


a) Absorption
b) Distribution
c) Metabolism
d) Excretion
e) Side effects
3) If a drug is 80% bound to blood elements or plasma proteins, what part is considered the free form?
a) 20%
b) 40%
c) 50%
d) 80%
e) 100%
4.1) Which of the following describes minimal effective concentration (MEC)?
a) The minimal drug plasma concentration that can be detected
b) The minimal drug plasma concentration to enter tissues
c) The minimal drug plasma concentration to interact with receptors
d) The minimal drug plasma concentration to produce effect
e) The minimal drug plasma concentration to reach therapeutic levels
4.2) If a patient misses three doses of their daily drug, which of the following (in general) is the best solution?
a) Take a 4x dose at the next dose time
b) Wait 3 more days (week total) then return to normal regimen
c) Do nothing and continue normal regimen
d) Setup an appointment to have the patient evaluated
e) Prescribe a higher dosage pill so missed doses will have less effect
4.3) Blood levels of a drug correlate to the effectiveness of that drug, such as with
pentazocine (Talwin) or phenobarbitol (Luminal).
a) True
b) False
5.1) Which of the following drug permeation mechanisms involves polar substances too large to enter cells by other means, such as iron or
vitamin B12?

a) Aqueous diffusion
b) Lipid diffusion
c) Carrier molecules
d) Endocytosis and exocytosis
5.2) Which of the following drug permeation mechanisms occurs across epithelial tight
junctions and is driven by a concentration gradient?
a) Aqueous diffusion
b) Lipid diffusion
c) Carrier molecules
d) Endocytosis and exocytosis
5.3) Which of the following drug permeation mechanisms uses the Henderson-
Hasselbalch equation for the ratio of solubility for the weak acid or weak base?
a) Aqueous diffusion
b) Lipid diffusion
c) Carrier molecules
d) Endocytosis and exocytosis
5.4) Which of the following drug permeation mechanisms is used for peptides, amino
acids, glucose, and other large or insoluble molecules?
a) Aqueous diffusion
b) Lipid diffusion
c) Carrier molecules
d) Endocytosis and exocytosis
5.5) Which of the following drug permeation mechanisms uses caveolae?
a) Aqueous diffusion
b) Lipid diffusion
c) Carrier molecules
d) Endocytosis and exocytosis
6.1) Using the Fick Law of Diffusion, how will flux change if membrane thickness is
doubled?
a) It will double
b) It will quadruple
c) It will halve
d) It will quarter
e) It will not change
6.2) Using the Fick Law of Diffusion, how will flux change if the permeability
coefficient is quadrupled?
a) It will double
b) It will quadruple
c) It will halve
d) It will quarter
e) It will not change
7.1) Which of the following is the amount of a drug absorbed per the amount
administered?
a) Bioavailability
b) Bioequivalence
c) Drug absorption
d) Bioinequivalence
e) Dosage
7.2) Which of the following is NOT needed for drug bioequivalence?
a) Same active ingredients
b) Same strength or concentration
c) Same dosage form
d) Same route of administration
e) Same side effects
7.3) For intravenous (IV) dosages, what is the bioavailability assumed to be?
a) 0%
b) 25%
c) 50%
d) 75%
e) 100%
7.4) Although morphine (Avinza, Oramorph SR, MS Contin) is well-absorbed when
administered orally (PO), how much of the drug is metabolized on its first pass through
the liver?
a) 90%
b) 70%
c) 50%
d) 30%
e) 10%
7.5) For a generic drug to be bioequivalent to an innovator drug (per FDA), it must be
measured in ____ of subjects to fall within ____ of the mean of the test population
bioavailability.
a) 50; 50
b) 80; 20
c) 20; 80
d) 95; 5
e) 5; 95
7.6) Using the FDA bioequivalence rule, how much variation could a generic drug
potentially have from an innovator and still be considered equivalent?
a) 100%
b) 20%
c) 40%
d) 60%
e) 80%
8.1) Which of the following is NOT a pharmacokinetic process?
a) Alteration of the drug by liver enzymes
b) Drug metabolites are removed in the urine
c) Movement of drug from the gut into general circulation
d) The drug causes dilation of coronary vessels
e) The drug is readily deposited in fat tissue
8.2) Which of the following can produce a therapeutic response? A drug that is:
a) Bound to plasma albumin
b) Concentrated in the bile
c) Concentrated in the urine
d) Not absorbed from the GI tract
e) Unbound to plasma proteins
8.3) Which of the following most correctly describes steroid hormones with respect to
their ability to gain access to intracellular binding sites?
a) They cross the cell membrane via aqueous pores
b) They have a high permeability coefficient
c) They are passively transported via membrane carriers
d) They require vesicular transport
e) Their transport requires the hydrolysis of ATP
3 – Pharmacokinetic Principles: pH and Drug Movement
1) Most drugs are either ____ acids or ____ bases.
a) Strong; Strong
b) Strong; Weak
c) Weak; Weak
d) Weak; Strong
2.1) Aspirin readily donates a proton in aqueous solutions and pyrimethamine readily
accepts a proton in aqueous solution. Thus, aspirin is a(b) ____ and pyrimethamine is
a(n) ____.
a) Acid; Base
b) Base; Acid
c) Acid; Acid
d) Base; Base
2.2) Given the equilibrium HA <=> A- + H+ (acid) and BH+ <=> B + H+ (base), in an
acid environment (low pH) the acid reaction will move to the ____ and the base reaction
will move to the ____.
a) Right; Left
b) Right; Right
c) Left; Right
d) Left; Left
3.1) What form of a drug is more lipid-soluble, and thus would remain trapped within a
compartment where the pH does not favor the lipid-soluble form?
a) Strong acid (A-)
b) Weak acid (A-)
c) Neutral (AH and B)
d) Weak base (BH+)
e) Strong base (BH+)
3.2) The lipid-soluble form of a base is ____ and the lipid-soluble form of an acid is
____.
a) Protonated; Protonated
b) Protonated; Unprotonated
c) Unprotonated; Unprotonated
d) Unprotonated; Protonated Pharmacology – Part 1 Quiz
Version: 16Oct2008 Page 8 of 42
4.1) If the pKa of Aspirin (acetylsalicylic acid) is 3.5 and the pH of the stomach is 2.5,
how much Aspirin is in the protonated species in the stomach and is this the amount
available for absorption?
a) ≈ 91%; Yes
b) ≈ 91%; No
c) ≈ 9%; Yes
d) ≈ 9%; No
4.2) What percentage of Aspirin would be ionized in the blood compartment (pH = 7.4)
assuming pH is 7.5 and Aspirin pKa is 3.5?
a) (10,000 – 1) / 1 = 99.99%
b) (100 – 1) / 1 = 99%
c) None
d) 1 / (100 – 1) = 0.9%
e) 1 / (10,000 – 1) = 0.009%
4.3) If the pH – pKa = -1, what percentage of weak base is nonionized?
a) 99
b) 90
c) 50
d) 10
e) 1
4.4) If the pH – pka = 2, what percentage of weak acid is nonionized?
a) 99
b) 90
c) 50
d) 10
e) 1
4.5) If pH > pKa, the drug is ____ and if pH < pKa, the drug is ____. An unprotonated
acid is ____ and a protonated base is ____.
a) Protonated; Unprotonated; Charged; Charged
b) Protonated; Unprotonated; Neutral; Neutral
c) Unprotonated; Protonated; Charged; Charged
d) Unprotonated; Protonated; Neutral; Charged
e) Unprotonated; Protonated; Charged; Neutral
5.1) Weak acids are excreted faster in ____ urine and weak bases are excreted faster in
____ urine.
a) Acidic; Alkaline
b) Alkaline; Acidic
c) Acidic; Neutral
d) Neutral; Alkaline
e) Alkaline; Neutral
5.2) A patient presents with an overdose of acidic Aspirin. The drug ____ can be given to
____ the pH of the urine and trap the Aspirin, preventing further metabolism.
a) NaHCO3; Increase
b) NaHCO3; Decrease
c) NH4Cl; Increase
d) NH4Cl; Decrease Pharmacology – Part 1 Quiz
Version: 16Oct2008 Page 9 of 42
5.3) A patient presents with an overdose of alkaline Codeine. The drug ____ can be given
to ____ the pH of the urine and trap the Codeine, preventing further metabolism.
a) NaHCO3; Increase
b) NaHCO3; Decrease
c) NH4Cl; Increase
d) NH4Cl; Decrease
6.1) The principle of drug manipulation for excretion of a drug out of the renal tubule can
be accomplished by:
a) Acidifying the urinary pH
b) Adjusting the urinary pH to protonate weakly acidic drugs
c) Adjusting the urinary pH to unprotonate weakly basic drugs
d) Adjusting the urinary pH to ionize the drug
e) By neutralizing the urinary pH
6.2) Aspirin is a weak organic acid with a pKa of 3.5. What percentage of a given dose
will be in the lipid-soluble form at a stomach pH of 1.5?
a) About 1%
b) About 10%
c) About 50%
d) About 90%
e) About 99%
6.3) For which of the following drugs is excretion most significantly accelerated by
acidification of the urine?
a) Weak acid with pKa of 5.5
b) Weak acid with pKa of 3.5
c) Weak base with pKa of 7.5
d) Weak base with pKa of 7.1
6.4) A patient diagnosed with type 2 diabetes is administered an oral dose of 0.1 mg
chloropropamide, an insulin secretagogue and weak acid with a pKa of 5.0. What is the
amount of this drug that could be absorbed from the stomach at pH 2.0?
a) 99.9 µg
b) 90 µg
c) 50 µg
d) 0.05 mg
e) 0.01 mg

4 – Pharmacokinetic Principles: Absorption


1) Bioavailability (F) is the fraction or percentage of administered drug that reaches the
systemic circulation via a given route as compared to what route?
a) Oral
b) IV (intravenous)
c) IO (intraosseous)
d) CSF (cerebrospinal fluid)
e) Whatever route attains the target drug concentration in plasma (CT)
2) What organ is responsible for metabolism in the “first pass effect”?
a) Brain
b) Heart
c) Kidney
d) Liver
e) Spleen
3.1) A patient is in the hospital and is stable on digoxin 0.175 mg IV qd (daily). How
much digoxin in mg. would you need to give your patient orally, given that the
bioavailability for oral digoxin tablets is 0.7?
a) (0.175 * 0.7) / (1.0) = 0.1225 mg
b) (0.175 * 1) / (0.7) = 0.25 mg
c) (0.175 + 0.7) / (1.0) = 0.875 mg
d) (0.175 + 1) / (0.7) = 1.67 mg
e) No change is necessary
3.2) Given a graph of plasma drug concentration versus time, what part of the graph
would be used to calculate bioavailability for a PO (oral) drug administration?
a) Maximum concentration
b) Steady concentration
c) Derivative of the curve (slope)
d) Integral of the curve (area underneath)
e) The curve is not used to calculate bioavailability
4.1) Which of the following routes of administration has a bioavailability of about 80-
100%, is usually very slow absorbing, and has prolonged duration of action?
a) IV (intravenous)
b) IM (intramuscular)
c) SQ (subcutaneous)
d) Rectal
e) Transdermal
4.2) Which of the following routers of administration is the most convenient, although
may have a bioavailability anywhere from 5-100%?
a) PO (oral)
b) IV (intravenous)
c) IM (intramuscular)
d) SQ (subcutaneous)
e) Transdermal
4.3) Which of the following enteral administration routes has the largest first-pass effect?
a) SL (sublingual)
b) Buccal
c) Rectal
d) Oral
4.4) Epithelial cells are connected by ____, which are tough to cross and materials often
must pass through the cells. Endothelial cells of blood vessels are connected by ____,
which proteins cannot cross but smaller drugs (MW 200-500) can.
a) Macular gap junctions; Tight junctions
b) Tight junctions; Macular gap junctions
c) Adherens junctions; Tight junctions
d) Tight junctions; Adherens junctions
e) Macular gap junctions; Adherens junctions
4.5) Which of the following administration routes is not often used, is painful, and has a
risk of infection and adhesion?
a) EPI (epidural)
b) IA (intraarterial)
c) IP (intraperitoneal)
d) IV (intravenous)
e) IO (intraosseous)
4.6) Which of the following is NOT an advantage of prolonged release medications?
a) Less frequent administration
b) Therapeutic effect overnight
c) Lower incidence of side effects
d) Patient compliance
e) More fluctuation in plasma concentration
4.7) What is the common location for the scopolamine motion sickness transdermal
patch?
a) Side of the hip
b) Chest
c) Over the deltoid muscle
d) Behind the ear
e) On the back of the neck
5 – Pharmacokinetic Distribution: Basics
1.1) Which of the following would receive drug slowly?
a) Liver
b) Brain
c) Fat
d) Muscle
e) Kidney
1.2) Which of the following is the least important for passage through capillary walls but
the most important for passage through the cell wall?
a) Molecular size
b) Lipid solubility
c) Diffusion constant
d) pH
e) pKa
1.3) Which of the following is the most important for movement through capillary walls?
a) Molecular size
b) Lipid solubility
c) Diffusion constant
d) pH
e) pKa
1.4) Which of the following locations would most trap a lipid soluble drug?
a) Blood
b) Intestines
c) Brain
d) Stomach
1.5) What type of drugs can cross the blood-brain barrier (BBB)?
a) Large and lipid-soluble
b) Large and lipid-insoluble
c) Small and lipid-soluble
d) Small and lipid-insoluble
2.1) Acidic drugs, such as phenytoin, bind primarily to which of the following plasma
proteins?
a) α1-fetoprotein (AFP)
b) GC Globulin
c) Albumin
d) α1-acid glycoprotein (AAG)
e) Transcortin
2.2) Basic drugs, such as lidocaine, bind primarily to which of the following plasma
proteins?
a) α1-fetoprotein (AFP)
b) Gc-Globulin (GcG)
c) Albumin
d) α1-acid glycoprotein (AAG)
e) Transcortin
3.1) A decrease in drug-protein binding will lead to which of the following?
a) Decrease in the unbound drug concentration
b) Increase in free drug
c) Increase in rate of drug elimination
d) Decrease in volume of distribution
3.2) A patient presents with acute-onset cirrhosis of the liver. They are found to have
hypoalbuminemia. In severe cirrhosis it is expected that AAG will be decreased, but the
patient presents with increased AAG due to the inflammatory response. Which of the
following is the most likely?
a) Increased acidic drug binding and increased basic drug binding
b) Increased acidic drug binding and decreased basic drug binding
c) Decreased acidic drug binding and increased basic drug binding
d) Decreased acidic drug binding and decreased basic drug binding
3.3) Which of the following is NOT a site of loss (where drug is not used)?
a) Fat
b) GI tract
c) Muscle
d) Site lacking receptors
4.1) Which of the following locations can accumulate lipid-soluble drugs, has little or no
receptors, and can hold distributed drugs like barbiturates?
a) Liver
b) Kidney
c) Brain
d) Fat
e) Fetus
4.2) Which of the following locations has high blood flow and is a site of excretion?
a) Liver
b) Kidney
c) Brain
d) Fat
e) Fetus
4.3) Anything affecting renal perfusion will affect drug delivery to the kidney, drug
excretion, and drug levels in the blood.
a) True
b) False
4.4) Which of the following can be treated with drugs due to a leaky area in the blood-
brain barrier near the medulla?
a) Seizures
b) Shivers
c) Diarrhea
d) Nausea
e) Vomitting
4.5) What is the approximate lag time for equilibration between maternal blood and fetal
tissues?
a) 20 mins
b) 40 mins
c) 1 hour
d) 2 hours
e) 6 hours
Match the body compartment with the volume, assuming a 70kg male patient:
5.1) Total body a) 4
5.2) Plasma b) 10
5.3) Interstitial c) 14
5.4) Extracellular d) 28
5.5) Intracellular e) 42
5.6) If protein plasma binding is decreased, how will volume of distribution be affected?
a) Increased
b) Decreased
c) Not changed
5.7) 400 mg of a drug is administered to a patient and the drug is later measured in
plasma to be 1 µg/ml. What is the apparent volume of distribution (Vd)?
a) 0.04 L
b) 0.4 L
c) 4 L
d) 40 L
e) 400 L
5.8) Elderly patients often have ____ muscle mass and thus a(n) ____ Vd.
a) More; Increased
b) More; Decreased
c) Less; Increased
d) Less; Decreased
5.9) Patients with ascites or edema would have ____ Vd for hydrophilic drugs, such as
gentamicin.
a) Increased
b) Decreased
c) Unchanged
6 – Pharmacokinetics: Drug Metabolism
1.1) Which of the following locations is the most likely for finding a free, unaltered drug?
a) Urine
b) Feces
c) Breast milk
d) Fat
e) Sweat

1.2) Most drugs are active in their ____ form and inactive in their ____ form.
a) Non-polar; Polar
b) Polar; Non-polar
c) Water-soluble; Lipid-soluble
d) Lipid-insoluble; Water-insoluble
e) Neutral; Neutral
2.1) Drug biotransformation phase I makes drugs ____ polar for metabolism and phase II
makes drugs ____ polar for excretion.
a) More; More
b) More; Less
c) Less; More
d) Less; Less
2.2) Which of the following is NOT a phase II substrate?
a) Glucuronic acid
b) Sulfuric acid
c) Acetic acid
d) Amino acids
e) Alcohol
3) Which of the following reactions is phase II and NOT phase I?
a) Oxidations
b) Reductions
c) Conjugations
d) Deaminations
e) Hydrolyses
4) Which of the following metabolically active tissues is the principle organ for drug
metabolism?
a) Skin
b) Kidneys
c) Lungs
d) Liver
e) GI Tract
5.1) Damage at which of the following locations would most affect the goals of phase II
biotransformation?
a) Skin
b) Kidneys
c) Lungs
d) Liver
e) GI Tract
Match the biotransformation reaction with the drug:
5.2) Hydroxylation of aromatic ring to increase polarity a) Codeine
5.3) N-dealkylation b) Morphine
5.4) Sulfoxidation c) Thioridazine
5.5) O-dealkylation d) Nicotine
5.6) N-oxidation e) Phenobarbitol
5.7) Side chain oxidation with -OH to increase polarity f) Pentobarbitol
5.8) Conversion to glutathione and reactive intermediate g) Acetaminophen
6.1) What is the goal of the P450 system (microsomes pinched off from endoplasmic
reticulum)?
a) Metabolism of substances
b) Detoxification of substances
c) Increasing pH of compartments containing substances
d) Decreasing pH of compartments containing substances
e) A & B
6.2) Regarding the microsomal drug metabolizing system, a patient with late stage
alcoholism and liver damage would have more ETOH available due to which of the
following concepts?
a) Increased induction
b) Decreased induction
c) Increased inhibition
d) Decreased inhibition
6.3) Regarding the microsomal drug metabolizing system, a patient who is a chronic user
of barbiturates would need more drug to produce the same effects due to which of the
following concepts?
a) Increased induction
b) Decreased induction
c) Increased inhibition
d) Decreased inhibition
6.4) Which of the following are the drugs that induce CYP 1A2 and the drugs that have
their metabolism induced by 1A2?
a) Carbamazepine & phenobarbitol; Theophyline & warfarin
b) Phenobarbitol & phenytoin ; Phenytoin & warfarin
c) Carbamazepine & phenytoin; Warfarin
d) Carbamazepine; Cyclosporine
6.5) Which of the following are the drugs that inhibit CYP 1A2 and the drugs that have
their metabolism inhibited by 1A2?
a) SSRIs; Phenytoin & warfarin
b) Amiodarone & cimetidine; Phenytoin & warfarin
c) Cimetidine, erythromycin, & grapefruit juice; Theophyline & warfarin
d) Cimetidine & erythromycin; Cyclosporine
6.6) Which of the following groups of people is the least likely to have biotransformation
effects due to altered hepatic function?
a) Infants

b) Adults

c) Elderly

d) Chronic alcoholics

e) Acetaminophen overdoses

6.7) In what location does amino acid conjugation of glycine (e.g. salicyclic acid) take

place?

a) Microsomal

b) Cytosol

c) Mitochondria

6.8) Where does acetylation conjugation (e.g. isoniazid) and sulfate conjugation (e.g.
acetaminophen) take place?

a) Microsomal

b) Cytosol

c) Mitochondria

6.9) Where does glucuronide conjucation (e.g. digoxin, bilirubin) take place?

a) Microsomal

b) Cytosol

c) Mitochondria

6.10) What is a result of conjugation of isoniazid via N-acetylation?

a) Detoxification of liver

b) Detoxification of kidneys

c) Detoxification of blood

d) Detoxification of urine

e) Hepatotoxicity

7 – Pharmacokinetics: Principles of Eliminations


1.1) One liter contains 1,000 mg of a drug. After one hour, 900 mg of the drug remains.
What is the clearance?
a) 100 mL
b) 100 mL/hr
c) 1 mg/ml
d) 100 mg
e) 1 mg/sec
1.2) To maintain a drug concentration at steady state, the dosing rate should equal the
elimination rate. Which of the following is true? (CL = Drug Clearance)
a) Dosing rate = CL + target concentration
b) Dosing rate = CL – target concentration
c) Dosing rate = CL * target concentration
d) Dosing rate = CL / target concentration
1.3) Which of the following is most useful in determining the rate of elimination of a
drug, in general?
a) Drug concentration in urine (renal elimination)
b) Drug concentration in stool (bilary elimination)
c) Drug concentration in blood
d) Drug concentration in brain
e) Drug oxidation rate
2.1) For first-order drug elimination, half life t(1/2) is ____ at two places on the curve
and a constant ____ is lost per unit time.
a) Equal; Amount
b) Equal; Percentage
c) Not equal; Amount
d) Not equal; Percentage
2.2) For first-order drug elimination, given the half-life equation of t(1/2) = (0.693 * Vd)
/ CL, how many half-lives would be necessary to reach steady state (≈95%) without a
loading dose?
a) 1 to 2
b) 2 to 3
c) 3 to 4
d) 4 to 5
e) 5 to 6
2.3) Which of the following is NOT a drug exhibiting zero-order elimination kinetics?
a) Aspirin
b) Morphine
c) Phenytoin
d) ETOH
2.4) For zero-order drug elimination, half-life t(1/2) is ____ at two places on the curve
and a constant ____ is lost per unit time.
a) Equal; Amount
b) Equal; Percentage
c) Not equal; Amount
d) Not equal; Percentage
2.5) If a drug with a 2-hour half life is given with an initial dose of 8 mcg/ml, assuming
first-order kinetics, how much drug will be left at 6 hours?
a) 8 mcg/ml
b) 4 mcg/ml
c) 2 mcg/ml
d) 1 mcg/ml
e) 0.5 mcg/ml
3.1) What are the units for steady-state concentration (Css), or infusion rate over
clearance?
a) mg/min
b) ml/min
c) mg/ml
d) ml/mg
e) min/mg
3.2) What percentage of the steady-state drug concentration is achieved at 3.3 * t(1/2)?
a) 10%
b) 25%
c) 50%
d) 75%
e) 90%
4.1) Increasing the rate of infusion changes the time necessary to reach the steady-state
concentration.
a) True
b) False
4.2) An injection of two units of a drug once-daily (qd) will yield the same steady-state
concentration as an injection of one unit of a drug twice-daily (bid).
a) True
b) False
5.1) Which of the following drugs would most likely need a loading dose to help reach
therapeutic levels?
a) Acetaminophen, t(1/2) = 2 h
b) Aspirin, t(1/2) = 15 m
c) Tetracycline, t(1/2) = 11 h
d) Digitoxin, t(1/2) = 161 h
e) Adenosine, t(1/2) = 10 s
5.2) A target concentration of 7.5 mg/L of theophylline is required for a 60 kg patient.
What is the loading dose, given the following: Vd = 0.5 L/kg, Cl = 0.04 L/kg/hr, t(1/2) =
9.3 hr?
a) 0.5 L/kg * 60 kg * 7.5 mg/L = 225 mg/h, infusion
b) 0.5 L/kg * 60 kg * 7.5 mg/L = 225 mg, bolus
c) 0.04 L/kg/hr * 60 kg * 7.5 mg/L = 18 mg/h, infusion
d) 0.04 L/kg/hr * 60 kg * 7.5 mg/L = 18 mg, bolus
5.3) A target concentration of 7.5 mg/L of theophylline is required for a 60 kg patient.
What is the steady state maintenance dose, given the following: Vd = 0.5 L/kg, Cl = 0.04
L/kg/hr, t(1/2) = 9.3 hr?
a) 0.5 L/kg * 60 kg * 7.5 mg/L = 225 mg/h, infusion
b) 0.5 L/kg * 60 kg * 7.5 mg/L = 225 mg, bolus
c) 0.04 L/kg/hr * 60 kg * 7.5 mg/L = 18 mg/h, infusion
d) 0.04 L/kg/hr * 60 kg * 7.5 mg/L = 18 mg, bolus

11 – Autonomic Pharmacology: Sympathetic Nervous System


1.1) The sympathetic nervous system (SNS) and parasympathetic nervous system are
divisions of which of the following?
a) Somatic nervous system division of peripheral nervous system
b) Somatic nervous system division of central nervous system
c) Autonomic nervous system division of peripheral nervous system
d) Autonomic nervous system division of central nervous system

1.2) Preganglionic sympathetic and parasympathetic fibers release ____, postganglionic


parasympathetic fibers release ____ (for muscarinic receptors), and
postganglionic sympathetic fibers release ____ (for adrenergic receptors).
a) ACh; ACh; NE
b) ACh; NE; ACh
c) NE; ACh; NE
d) NE; NE; ACh
1.3) Which of the following adrenergic receptors is most commonly found pre-synaptic?
a) Alpha 1 receptors
b) Alpha 2 receptors
c) Beta 1 receptors
d) Beta 2 receptors
e) Beta 3 receptors
1.4) Which of the following describes the result of adrenal medulla stimulation?
a) Mass parasympathetic discharge, 85:15 ratio of epi:norepi
b) Mass parasympathetic discharge, 15:85 ratio of epi:norepi
c) Mass sympathetic discharge, 85:15 ratio of epi:norepi
d) Mass sympathetic discharge, 15:85 ratio of epi:norepi
Match the sympathetic response with the receptor:
1.5) Increased lipid breakdown a) α1
1.6) Peripheral vasoconstriction b) β1
1.7) Increased heart rate and blood pressure c) β2
1.8) Bronchial dilation, coronary dilation, glucose conversion d) β3
1.9) What amino acids is converted into catecholamines (NE, Epi, Dopamine)?
a) Alanine
b) Proline
c) Lysine
d) Tyrosine
e) Valine
1.10) Which of the following is transported into vesicles via the vesicular monoamine
transporter (VMAT), uptake 2, a proton antiporter?
a) Epinephrine
b) Norepinephrine
c) Dopamine
1.11) Which of the following is co-stored and co-released with ATP?
a) Epinephrine
b) Norepinephrine
c) Dopamine
1.12) Which of the following form varicosities or en passant synapses, with the arrival of
an action potential leading to Ca++ influx and exocytosis?
a) Presynaptic sympathetic
b) Presynaptic parasympathetic
c) Postsynaptic sympathetic
d) Postsynaptic parasympathetic
2.1) Which of the following methods of terminating axon response is NOT a target for
drug action?
a) Reuptake via NE transporter (NET): Uptake 1
b) Metabolism of NE of inactive metabolite
c) NE diffusion away from synaptic cleft
2.2) NET is a symporter of what ion?
a) K+
b) Ca++
c) Cl-
d) Na+
e) Mg++
2.3) Which of the following is recycled via VMAT into vesicles after response
termination?
a) NE
b) L-DOPA
c) NET
d) EPI
e) DOPGAL
2.4) Which of the following is broken down by MAO-B (monoamine oxidase) more than
the others?
a) Serotonin (5-HT)
b) Norepinepherine (NE)

c) Dopamine (DA)
2.5) Where is the cytosolic catecholamine metabolizing enzyme catechol-O-methyl
transferase (COMT) primarily found?
a) Liver
b) GI tract
c) Placenta
d) Blood platelets
3.1) Which of the following receptor subtypes relaxes smooth muscle and causes liver
glycogenolysis and gluconeogenesis?
a) α1 (Gq/Gi/Go)
b) α2 (Gi/Go)
c) β1 (Gs)
d) β2 (Gs)
e) β3 (Gs)
3.2) Which of the following receptor subtypes causes vascular smooth muscle contraction
and genitourinary smooth muscle contraction?
a) α1 (Gq/Gi/Go)
b) α2 (Gi/Go)
c) β1 (Gs)
d) β2 (Gs)
e) β3 (Gs)
3.3) Which of the following receptor subtypes increases cardiac chronotropy (rate) and
inotropy (contractility), increases AV-node conduction velocity, and increases rennin
secretion in renal juxtaglomerular cells?
a) α1 (Gq/Gi/Go)
b) α2 (Gi/Go)
c) β1 (Gs)
d) β2 (Gs)
e) β3 (Gs)
3.4) Which of the following receptor subtypes decreases insulin secretion from pancreatic
β-cells, decreases nerve cell norepinephrine release, and contracts vascular smooth
muscle?
a) α1 (Gq/Gi/Go)
b) α2 (Gi/Go)
c) β1 (Gs)
d) β2 (Gs)
e) β3 (Gs)
4.1) What type(s) of second messenger(s) interact with adenylyl cyclase?
a) α1
b) α2
c) β
d) β & α1
e) β & α2
4.2) What type(s) of second messenger(s) are associated with phospholipase C (PLC)?
a) α1
b) α2
c) β
d) β & α1
e) β & α2
4.3) Which of the following adrenergic receptor activation mechanisms is involved with
ephedrine, amphetamine, and tyramine?
a) Direct binding to the receptor
b) Promoting release of norepinephrine
c) Inhibiting reuptake of norepinephrine
d) Inhibiting inactivation of norepinephrine
4.4) Which of the following adrenergic receptor activation mechanisms is involved with
MAO inhibitors?
a) Direct binding to the receptor
b) Promoting release of norepinephrine
c) Inhibiting reuptake of norepinephrine
d) Inhibiting inactivation of norepinephrine
4.5) Which of the following adrenergic receptor activation mechanisms is involved with
tricyclic antidepressants and cocaine?
a) Direct binding to the receptor
b) Promoting release of norepinephrine
c) Inhibiting reuptake of norepinephrine
d) Inhibiting inactivation of norepinephrine
4.6) Which of the following is NOT true of catecholamines?
a) Non-polar
b) Cannot cross the blood-brain barrier
c) Cannot be used as an oral drug
d) Have brief duration
e) MAO and COMT act rapidly
Match the catecholamine with the receptor(s):
4.7) Isoproterenol a) α & β
4.8) Dobutamine b) β
4.9) Norepinepherine c) β1
4.10) Dopamine d) D1 & D2
4.11) Epinepherine
4.12) The basic structure of a catecholamine involves a catechol ring and which of the
following types of amines?
a) Methyl amine
b) Ethyl amine
c) Butyl amine
d) Tert-butyl amine
e) Propyl amine
Match the noncatecholamines with the receptor agonist:
4.13) Clonidine a) α1-agonist
4.14) Metaproterenol, terbutaline, ritodine b) α2-agonist

4.15) Phenylephrine c) β2-agonist

5.1) Which of the following is a long-acting (oral) α1-agonist and not a short-acting
(nasal spray, ophthalmic drops) α1-agonist?
a) Phenylephrine
b) Oxymetazoline
c) Tetrahydrazaline
d) Pseudoephedrine
5.2) Which of the following would NOT be used as a topical vasoconstrictor for a patient
with epistaxis (nasal pack soaked in drug)?
a) Phenylephrine
b) Epinepherine
c) Oymetazoline
d) Isoproterenol
5.3) α1 drugs can be given with local anesthetics to vasoconstrictor and decrease blood
flow to the side of administration. Which of the following should not be given above the
web space?
a) Phenylephrine
b) Epinephrine
c) Methoxamine
5.4) Which of the following is the α1 drug of choice (DOC) for retinal exams and
surgery, giving mydiasis (dilation of iris)?
a) Ephedrine
b) Epinepherine
c) Oymetazoline
d) Isoproterenol
e) Phenylephrine
5.5) α2-agonists are only approved for hypertension and work by decreasing sympathetic
tone and increasing vagal tone. Which of the following is NOT a α2-agonist?
a) Clonidine
b) Methyldopa
c) Guanabenz
d) Guanfacine
e) Epinephrine
5.6) At the adrenergic synapse, what does α2 do?
a) Stimulates NE release
b) Inhibits NE release
c) Stimulates ACh release
d) Inhibits ACh release
5.7) Which of the following agonists would be used for asthma patients or to delay
premature labor?
a) α2-agonist
b) α1-agonist
c) β3-agonist
d) β2-agonist
e) β1-agonist
5.8) Which of the following agonists would be used for cardiogenic shock, cardiac arrest,
heart block, or heart failure?

a) α1-agonist
b) α2-agonist
c) β1-agonist
d) β2-agonist
e) β3-agonist
5.9) Which of the following is NOT a β2-agonist?
a) Terbutaline
b) Ritodrine
c) Metaproterenol
d) Albuterol
e) Phenylepherine
5.10) β2 stimulation leads to an increase in the cellular uptake of what ion, and thus a
decrease in plasma concentration of that ion?
a) K+
b) Ca++
c) Cl-
d) Na+
e) Mg++
5.11) Dopamine receptor activation (D1) dilates renal blood vessels at low dose. At
higher doses (treatment for shock), which of the following receptor is activated?
a) α1
b) α2
c) β1
d) β2
e) β3
5.12) Which of the following responses to sympathetic stimulation would prevent
receptors from being couples with G-proteins?
a) Sequestration
b) Down-regulation
c) Phosphorylation
5.13) Which of the following is the action of the indirect-acting sympathomimetic drug
cocaine?
a) Stimulator of NET (uptake 1)
b) Inhibitor of NET (uptake 1)
c) Stimulator of VMAT (uptake 2)
d) Inhibitor of VMAT (uptake 2)
5.14) Tricyclic antidepressants (TCAs) have a great deal of side effects. Which of the
following is the action of TCAs?
a) Stimulator of NET (uptake 1)
b) Inhibitor of NET (uptake 1)
c) Stimulator of VMAT (uptake 2)
d) Inhibitor of VMAT (uptake 2)
5.15) Which of the following is NOT a mixed sympathomimetic?
a) Amphetamine
b) Methamphetamine
c) Ephedrine
d) Phenylepherine
e) Pseudoephedrine
5.16) Prior to an operation to remove a pheochromocytoma (neuroendocrine tumor of the
medulla of the adrenal glands), which of the following should be given to the patient?
a) α-agonist
b) α-blocker
c) β-agonist
d) β-blocker
5.17) Which of the following is NOT an indication for β-blocker therapy?
a) Hypotension
b) Angina pectoris
c) Arrhythmias
d) Myocardial infarction
e) Glaucoma
5.18) Which of the following β-blockers is used for decreasing aqueous humor secretions
from the ciliary body?
a) Propranolol
b) Nadolol
c) Carvedilol
d) Timolol
e) Metoprolol
5.19) Which of the following is NOT considered cardioselective?
a) Metoprolol
b) Atenolol
c) Esmolol
d) Carvedilol
5.20) Blocking α2 presynaptic receptors will do which of the following?
a) Stimulate NE release
b) Inhibit NE release
c) Stimulate DA release
d) Inhibit DA release
5.21) Which of the following drugs irreversibly damages VMAT?
a) Tyramine
b) Guanethidine
c) Reserpine
d) Propranolol
e) Epinepherine
6.1) Which of the following is the most likely to occur with parenteral administration of a
α1-agonist drug?
a) Hypotension
b) Hypertension
c) Tissue necrosis
d) Vasodilation
e) Lipolysis
6.2) Which of the following agonists can have dose-related withdrawal syndrome if the
drug is withdrawn too quickly, leading to rebound hypertension?

a) α1-agonist
b) α2-agonist
c) β1-agonist
d) β2-agonist
e) β3-agonist
6.3) Which of the following agonists can have sedation and xerostomia (dry mouth) in
50% of patients starting therapy, sexual dysfunction in males, nauseas, dizziness, and
sleep disturbances?
a) α1-agonist
b) α2-agonist
c) β1-agonist
d) β2-agonist
e) β3-agonist
6.4) Which of the following agonists can cause hyperglycemia in diabetics?
a) α2-agonist
b) α1-agonist
c) β3-agonist
d) β2-agonist
e) β1-agonist
6.5) Angina pectoris, tachycardia, and arrhythmias are possible adverse effects of which
of the following agonists?
a) α2-agonist
b) α1-agonist
c) β3-agonist
d) β2-agonist
e) β1-agonist
6.6) If a patient is taking MAO inhibitors and ingests tyramine (red wine, aged cheese),
which of the following acute responses is most likely? (sympathomimetic)
a) Stimulation of NE release
b) Inhibition of NE release
c) Stimulation of ACh release
d) Inhibition of ACh release
e) No response due to MAO inhibitor
6.7) Which of the following occurs acutely, leading to a false neurotransmitter, with
increased guanethidine? (sympathomimetic)
a) Stimulation of NE release
b) Inhibition of NE release
c) Stimulation of ACh release
d) Inhibition of ACh release
6.8) Major adverse affects of the α1 blockade include reflex tachycardia and which of the
following?
a) Orthostatic tachycardia
b) Orthostatic bradycardia
c) Orthostatic hypertension
d) Orthostatic hypotension

e) Increased cardiac output


6.9) Which of the following effects would be intensified with the α2 blockade?
a) Reflex tachycardia
b) Reflex bradycardia
c) Orthostatic hypertension
d) Orthostatic hypotension
e) Platelet clotting
6.10) Which of the following is NOT an adverse affect of the β1 blockade?
a) Bradycardia
b) Decreased cardiac output
c) AV node block
d) Increased arrhythmias
e) Heart failure
6.11) Which of the following is the most severe adverse effect that has been associated
with sudden termination of β1-blockers?
a) Atrial fibrillation
b) Reflex bradycardia
c) Syncope (fainting)
d) Angina
e) Sudden death
6.12) Which of the following groups of patients is most at risk for adverse effect seen in
β2-blockers?
a) Asthmatics
b) Congestive heart failure patients
c) Trauma patients
d) Diabetics
e) Patients with deep vein thromboses (DVTs)
6.13) Which of the following can be detrimental in diabetics and also can lead to masking
of tachycardia, which is indicative of hypoglycemia?
a) α1-blocker
b) α2-blocker
c) β1-blocker
d) β2-blocker
e) β3-blocker

Item Number: 1684 correct answer: 5 category: Analgesics


1. A child has ingested an unknown substance and has evidence of
respiratory depression. This symptom is usually found with poisoning
due to:
1. amphetamines
2. atropine
3. mushrooms
4. kerosene
5. opioids

Item Number: 2534 correct answer: 3 category: Analgesics


2. The use of methadone in the treatment of heroin addiction continues
to be controversial. It would therefore be advantageous to find a substance
with the beneficial effects of methadone, but without its undesirable
characteristics. The correct statement is that:
1. methadone is not physically addicting and therefore very useful in
treating heroin addicts
2. the withdrawal syndrome of methadone is of shorter duration than that of
heroin
3. propoxyphene may successfully suppress the withdrawal syndrome in
heroin-addicted individuals
4. although propoxyphene does block heroin withdrawal, it itself is not
physically addicting
5. propoxyphene may be successfully substituted for heroin in the addicted
individual and abruptly discontinued after three to four weeks without
signs of an abstinence syndrome
Item Number: 2799 correct answer: 3 category: Analgesics
3. Chronic renal damage resulting from the ingestion of analgesics has
been suggested. The FALSE statement is:
1. phenacetin has been implicated
2. salicylates have been implicated
3. meperidine has been implicated
4. acetaminophen is a metabolite of phenacetin
5. combinations may be more harmful

Item Number: 3460 correct answer: 4 category: Analgesics


4. A 20-month old infant is brought to the emergency room with fever,
vomiting, stupor, and hyperpnea of 12 hours’ duration. His leukocyte count
is 6,000/cu mm. The chest roentgenogram is clear. Urinalysis shows albuminuria,
a positive test for reducing substance, and acetonuria. Ferric chloride added
to the boiled acidified urine shows a persistent purple color. The most likely
diagnosis is:
1. acute glomerulonephritis
2. diabetic acidosis
3. acute bacterial meningitis
4. salicylate poisoning
5. phenothiazine poisoning

Item Number: 3870 correct answer: 5 category: Analgesics


5. The gastric mucosa has the important ability to prevent movement of gastric
acid from the stomach lumen into the gastric wall. Some diseases and drug
regiments have been implicated as causes of increased gastric mucosal
permeability to hydrogen ion. Drugs which may increase gastric wall permeability
include:
1. erythromycin
2. indomethacin
3. nitrofurantoin
4. aspirin
5. 2,4

Item Number: 3871 correct answer: 4 category: Analgesics


6. In normal patients, the so-called gastric mucosa barrier protects the
mucosal lining from back diffusion of hydrogen ions and subsequent destruction.
In some patients even small changes in the gastric barrier allows back diffusion
of significant hydrogen ion and subsequent destructive action. Which of the
following pharmacologic agents have been implicated as causes of gastric
barrier breakdown?
1. caffeine
2. ethanol
3. aspirin
4. 2,3
5. All of the above

Item Number: 3907 correct answer: 5 category: Analgesics


7. Patients with normal platelet counts and normal bleeding time may
still bleed severely as a result of aspirin ingestion prior to a dental or
surgical procedure. The aspirin interference with normal platelet function may
last as long as:
1. 4 hours
2. 12 hours
3. 2 days
4. 5 days
5. 7 days

Item Number: 4099 correct answer: 3 category: Analgesics


8. The appropriate antidote in the treatment of pentazocine overdosage is:
1. nalorphine
2. levallorphan
3. naloxone
4. Any of the above
5. None of the above

Item Number: 4587 correct answer: 4 category: Analgesics


9. Acute hemorrhagic gastritis is one of the most frequent causes of severe
upper gastrointestinal bleeding. It is frequently related to recent ingestion of
ethanol and/or aspirin and may be life-threatening. Bleeding secondary to aspirin
is mainly due to:
1. inhibition of gastric prostaglandin synthesis
2. decreased renal excretion of the salicylate with attendant longer half-
life in serum
3. back diffusion of hydrogen ions across the gastric mucosa
4. 1,3
5. None of the above

Item Number: 4639 correct answer: 3 category: Analgesics


10. Many stimuli may cause vomiting. The chemoreceptor trigger zone of the
central nervous system is:
1. located in the cerebral cortex
2. stimulated in all forms of vomiting
3. stimulated by morphine and its congeners
4. All of the above
5. 1,3

Item Number: 5275 correct answer: 1 category: Analgesics


11. A pharmacologic agent which has the potential to cause increased
biliary tree pressure is:
1. morphine
2. warfarin
3. phenytoin
4. acetazolamide
5. carbon tetrachloride

Item Number: 6260 correct answer: 4 category: Analgesics


12. Many commonly utilized medications are subject to abuse. Normally nontoxic
medications, if ingested in excessive amounts, may cause end-organ damage.
Analgesic nephropathy has been associated with:
1. prolonged abuse of phenacetin
2. prolonged abuse of phenacetin-aspirin combinations
3. acetaminophen derived from phenacetin
4. All of the above
5. 1,3

Item Number: 6765 correct answer: 4 category: Analgesics


13. All of the following statements regarding acetaminophen toxicity are
true EXCEPT:
1. acetaminophen is the principal metabolic product of phenacetin
2. an overdose of 10 grams or more may produce hepatic necrosis in adults
3. acetaminophen overdose may produce transient azotemia or renal failure
4. forced diuresis may be a useful form of therapy for acetaminophen overdose
5. liver pathology is centrilobular and midzonal necrosis

Item Number: 9774 correct answer: 1 category: Analgesics


14. Opioid analgesics are sometimes associated with the production of
pulmonary disease. The most common pulmonary complication after oral ingestion is:
1. pulmonary edema
2. interstitial fibrosis
3. pulmonary calcification
4. bronchoconstriction
5. pleural effusion

Item Number: 6707 correct answer: 1 category: Analgesics


15. Characteristically observed in individuals following acute overdose of
opioids.
A. pinpoint pupils
B. depressed respiration
C. coma
D. convulsions
1. A,B,C
2. A,C
3. B,D
4. D only
5. All of the above

Item Number: 6708 correct answer: 2 category: Analgesics


16. TRUE statement concerning the actions of opioids on the secretion of pituitary
hormones.
A. suppress the secretion of luteinizing hormone and thyrotropin
B. reduce the release of prolactin
C. act as a stimulus for ADH secretion
D. inhibit the secretion of ACTH
1. A,B,C
2. A,C
3. B,D
4. D only
5. All of the above

Item Number: 6728 correct answer: 4 category: Analgesics


17. The duration of analgesia is one important characteristic which differentiates
one opioid from another. Which opioid possesses the shortest duration of
analgesia?
1. morphine
2. hydromorphone
3. codeine
4. meperidine
5. methadone

Item Number: 6733 correct answer: 5 category: Analgesics


18. Which of the following possess(es) some antagonist activity at opioid
receptors?
A. naloxone
B. pentazocine
C. butorphanol
D. nalorphine
1. A,B,C
2. A,C
3. B,D
4. D only
5. All of the above

Item Number: 6740 correct answer: 1 category: Analgesics


19. TRUE statements concerning the pharmacologic properties of salicylates
include:
A. high dose aspirin therapy can lower the serum urate concentration
B. aspirin is metabolized by a combination of 1st and zero order
kinetics
C. aspirin exerts its actions primarily by inhibition of
cyclooxygenase
D. aspirin overdose causes significant hepatic toxicity if ingested
in sufficient quantities
1. A,B,C
2. A,C
3. B,D
4. D only
5. All of the above

Item Number: 6754 correct answer: 2 category: Analgesics


20. The opioid expected to have the shortest duration of action following
subcutaneous administration of equianalgesic doses:
1. hydromorphone
2. meperidine
3. methadone
4. morphine
5. codeine

Item Number: 6756 correct answer: 2 category: Analgesics


21. A patient you follow in clinic has a well-known heroin abuse problem. Drugs
which could potentially prevent an abstinence withdrawal syndrome during
hospitalization include:
A. morphine
B. nalbuphine
C. methadone
D. butorphanol
1. A,B,C
2. A,C
3. B,D
4. D only
5. All of the above

Item Number: 6760 correct answer: 1 category: Analgesics


22. TRUE statement concerning the pharmacologic effects of salicylates include:
A. salicylates are thought to exert their activity at least partially
by inhibiting prostaglandin synthetase
B. high-dose salicylate therapy (> 5 grams/day) lowers the serum uric
acid concentration
C. the effect of salicylates upon platelet aggregation is
irreversible unlike that of other nonsteroidal anti-inflammatory
drugs
D. salicylate overdose is potentially fatal; however, prompt
administration of acetylcysteine will avert this danger
1. A,B,C
2. A,C
3. B,D
4. D only
5. All of the above

Item Number: 6990 correct answer: 5 category: Analgesics


23. The analgesic most apt to produce dysphoria:
1. morphine
2. meperidine
3. methadone
4. codeine
5. pentazocine

Item Number: 6998 correct answer: 1 category: Analgesics


24. Morphine’s affects the eye by:
1. producing miosis through an action on the oculomotor nerve
2. producing mydriasis through an action on the sympathetic system
3. decreasing pupillary responses to light
4. directly acting on the smooth muscles of the iris
5. directly acting on extrinsic muscles of the eye

Item Number: 7004 correct answer: 3 category: Analgesics


25. Aspirin is a nonopioid analgesic which is thought to work by inhibiting:
A. prostaglandin reductase
B. prostaglandin synthetase
C. thromboxane synthetase
D. cyclooxygenase
1. A,B,C
2. A,C
3. B,D
4. D only
5. All of the above
Item Number: 7006 correct answer: 3 category: Analgesics
26. Aspirin may be fatal if taken in sufficient quantity. The syndrome of acute
salicylate overdose in children is characterized by:
A. marked hypothermia secondary to an antipyretic effect
B. fever
C. peripheral edema
D. disturbance in acid-base and electrolyte balance
1. A,B,C
2. A,C
3. B,D
4. D only
5. All of the above

Item Number: 11217 correct answer: 3 category: Analgesics


27. Acetaminophen has been used as a safe and effective analgesic/ antipyretic
agent for over 80 years. Since it may be purchase without a prescription, it
is readily available and as such the recommended dose may be exceeded. The
primary toxicity leading to death from an acetaminophen overdose is:
1. papillary necrosis and chronic interstitial nephritis
2. pancytopenia
3. hepatocellular necrosis
4. myocarditis
5. hemolytic anemia

Item Number: 11645 correct answer: 4 category: Analgesics


28. The pharmacologic effects of morphine include all EXCEPT:
1. behavioral changes
2. miosis
3. respiratory depression
4. diarrhea
5. postural hypotension

Item Number: correct answer: 2 category: Anesthetics


1. All of the following factors influence the rate of induction of anesthesia
with an inhaled anesthetic EXCEPT:
1. aqueous solubility of the anesthetic
2. patient history of malignant hyperthermia
3. ventilation rate
4. tension of gas administration
5. pulmonary blood flow rate
For questions 2 – 9, choose the correct answer from the following list. Each
answer may be used once, more than once, or not at all.

Anesthetic Partition Minimum Alveolar


Coefficient Concentration (MAC)
1. halothane 2.30 0.75
2. enflurane 1.80 1.68
3. nitrous oxide 0.47 105.00
4. isoflurane 1.40 1.15
5. methoxyflurane 12.00 0.16
Item Number: correct answer: 3 category: Anesthetics
2. The blood:gas partition coefficient is the ratio of anesthetic concentration
in blood compared to gas phase. Greater solubility in blood results in slower
onset of anesthesia and higher partition coefficient value. Which anesthetic
has the fastest onset of action?

Item Number: correct answer: 5 category: Anesthetics


3. The minimum alveolar concentration (MAC) is the concentration of anesthetic at
1 atmosphere of pressure that produces immobility in 50% of patients exposed
to a noxious stimulus. Which anesthetic is most potent by this measure?
Item Number: correct answer: 3 category: Anesthetics
4. A good analgesic at subanesthetic doses

Item Number: correct answer: 3 category: Anesthetics


5. The anesthetic that may be most safely used in a patient with a history of
malignant hyperthermia?

Item Number: correct answer: 1 category: Anesthetics


6. Anesthetic associated with the highest incidence of hepatitis.

Item Number: correct answer: 2 category: Anesthetics


7. The anesthetic which should be avoided in patients with a seizure disorder
because tonic-clonic seizures are associated with its use.

Item Number: correct answer: 4 category: Anesthetics


8. The halogenated anesthetic which is advantageous in patients with
cardiovascular disease because it maintains cardiac output, produces systemic
and coronary vasodilation, and catecholamine dependent arrhythmias are
uncommon.

Item Number: correct answer: 5 category: Anesthetics


9. This anesthetic has become obsolete due to its potential for causing
nephrotoxicity.

For questions 10 – 13, choose the correct answer from the following list. Each
answer may be used once, more than once, or not at all.
1. thiopental
2. propofol
3. ketamine
4. etomidate
5. fentanyl
Item Number: correct answer: 4 category: Anesthetics
10. The intravenous anesthetic with excitatory effects on the central nervous
system but produces the least cardiovascular disturbance among intravenous
anesthetics.

Item Number: correct answer: 2 category: Anesthetics


11. Anesthetic that must be administered in a lipid diluent due to poor aqueous
solubility.

Item Number: correct answer: 3 category: Anesthetics


12. In contrast to most anesthetics, this agent produces cardiac stimulation,
resulting in increased blood pressure, heart rate and cardiac output.

Item Number: correct answer: 5 category: Anesthetics


13. Anesthetic frequently used for analgesia during surgery. Large doses induce or
maintain anesthesia but mechanical ventilation required due to profound
central respiratory depression.

Item Number: correct answer: 4 category: Anesthetics


14. True statement regarding thiopental:
1. causes increased EEG activity
2. contraindicated in a patient with epilepsy
3. provides analgesia
4. associated with dose-dependent respiratory depression
5. provides adequate skeletal muscle relaxation alone

Item Number: correct answer: 2 category: Anesthetics


15. False statement regarding propofol as an anesthetic:
1. pharmacokinetic half life does NOT correlate with duration of central
nervous system depression
2. provides adequate skeletal muscle relaxation alone
3. vasodilation may produce a decline in blood pressure. Can be minimized
by decreasing dose.
4. safe alternative for patients predisposed to malignant hyperthermia
5. high lipophilicity provides rapid and smooth onset, dose titratability,
and rapid recovery from anesthesia
Item Number: correct answer: 1 category: Anesthetics
16. FALSE statement regarding ketamine as an anesthetic:
1. contraindicated in a patient with asthma because it may cause
bronchoconstriction
2. recovery from anesthesia associated with emergence phenomenon of
hallucinations and vivid dreams
3. produces dissociative anesthetic state, whereby patient appears awake
but is unconscious
4. produces analgesia

Item Number: correct answer: 4 category: Anesthetics


17. Correct statements regarding etomidate include all EXCEPT:
1. inhibits adrenocortical function
2. no adverse effects from adrenocortical inhibition during short-term use
3. produces minimal cardiovascular effects
4. potent hypnotic with excellent analgesic properties
5. safe alternative for a patient with unstable cardiovascular status

Item Number: correct answer: 3 category: Anesthetics


18. TRUE statement regarding midazolam as an adjunct to anesthesia:
1. provides analgesia
2. provides adequate skeletal muscle relaxation alone
3. useful for ameliorating seizure activity
4. produces cardiovascular stimulation
5. contraindicated in patients with asthma or COPD

Item Number: correct answer: 2 category: Anesthetics


19. FALSE statement regarding bupivacaine, a local anesthetic with a molecular
structure that includes an amide linkage.
1. undergoes hepatic metabolism
2. rapidly hydrolyzed by plasma esterase enzymes
3. contraindicated in a patient with a history of allergy to lidocaine
4. higher dose necessary to induce epidural anesthesia than spinal
anesthesia
5. higher risk of systemic side effects when used for epidural anesthesia
compared to spinal anesthesia

Item Number: correct answer: 4 category: Anesthetics


20. FALSE statement regarding local anesthetics:
1. produce effects by inhibiting voltage gated sodium channels
2. inhibition of sodium channels enhanced by repetitive depolarizations
3. small nerve fibers more susceptible than large nerve fibers
4. all sensory functions of a nerve affected equally at onset of anesthesia
5. vasoconstriction prolongs duration of action

Item Number: correct answer: 5 category: Anesthetics


21. Treatment of malignant hyperthermia includes all of the following EXCEPT:
1. cessation of anesthetic and changing of rubber tubing to anesthesia
machine
2. administration of dantrolene sodium 1mg/kg by rapid intravenous infusion
until symptoms subside
3. hyperventilation of patient with 100% oxygen
4. administration of fluids and diuretics to control myoglobinemia and
hyperkalemia
5. lidocaine is indicated for arrhythmias

Item Number: 9623 correct answer: 5 category: Inotropes/Vasodilators


1. Digitalis glycosides slows the heart rate in patients with systolic
dysfunction as a result of all of the following EXCEPT:
1. direct action of digitalis on the AV node to slow conduction
2. indirect effect of enhanced vagal tone on AV node
3. enhanced parasympathetic outflow from the CNS through an indirect
mechanism
4. diminished sympathetic tone which is high as a compensatory mechanism in
heart failure
5. enhanced responsiveness of the SA and AV node to norepinephrine through
an indirect mechanism

Item Number: 11356 correct answer: 1 category: Inotropes/Vasodilators


2. Which of the following drug-mechanism of action pairs is properly matched?
1. digoxin – inhibits Na+ – K+ ATPase pump
2. dobutamine – inhibits troponin I
3. amrinone – inhibits troponin I
4. dopamine – inhibits troponin I
5. isoproterenol – inhibits 1 and 2 receptors
Item Number: 1161 correct answer: 2 category: Inotropes/Vasodilators
3. The rapid heart rate sometimes seen after nitroglycerin administration is best
explained by:
1. a direct positive chronotropic effect on the myocardium
2. reflex sympathetic discharge due to a fall in systemic blood pressure
3. the ability of nitroglycerin to release norepinephrine from sympathetic
nerve endings
4. a decrease in intracranial pressure

Item Number: 1158 correct answer: 3 category: Inotropes/Vasodilators


4. The effectiveness of digitalis in the treatment of atrial flutter is primarily
due to its ability to:
1. slow the rate of firing of the S-A node
2. exert an atropine-like effect on the A-V node
3. slow conduction through the A-V node
4. decrease the refractory period through the A-V node
5. decrease the rate of conduction through atrial muscle

Item Number: 3828 correct answer: 5 category: Inotropes/Vasodilators


5. When digitalis therapy is initiated, serious cardiac arrhythmias may be caused
by a deficiency of:
1. Na+
2. K+
3. Cl-
4. Mg++
5. 2,4

Item Number: 3961 correct answer: 3 category: Inotropes/Vasodilators


6. The cardiac glycosides are used in the treatment of congestive heart failure
and atrial fibrillation. Given the differences between the cardiac glycosides
in their length of action and organs of major excretion, the correct statement
concerning the use of digitoxin is:
1. gastrointestinal absorption of digitoxin is incomplete and highly
variable within patients
2. digitoxin is excreted primarily unchanged in the kidney with a half life
of about 24 hours
3. digitoxin is highly protein bound in the serum
4. digitoxin has a much higher incidence of toxicity in patients with renal
disease
5. digitoxin is only available as an oral preparation
Item Number: 11384 correct answer: 4 category: Inotropes/Vasodilators
7. Digitalis toxicity manifested by premature ventricular contractions maybe
treated with all of the following EXCEPT:
1. lidocaine
2. digitalis-specific immune FAB antibody
3. phenytoin
4. quinidine
5. All are correct

Item Number: 11359 correct answer: 2 category: Inotropes/Vasodilators


8. The correct statement regarding digitalis is:
1. in normal individuals, digitalis increases cardiac output
2. in normal individuals, digitalis increases myocardial oxygen consumption
3. in normal individuals, digitalis reduced total peripheral resistance in
response to enhanced myocardial contractility
4. in normal individuals, digitalis depresses myocardial contractility
5. in normal individuals, digitalis increases heart rate

Item Number: 3718 correct answer: 1 category: Inotropes/Vasodilators


9. A routine electrocardiogram reveals a borderline delay in PR interval, sagging
left ventricular ST segments and a shortened RT interval. The most likely
explanation would be:
1. digitalis effect
2. acute sub-endocardial ischemia
3. hypokalemia
4. hypocalcemia
5. myxedema

Item Number: 11367 correct answer: 4 category: Inotropes/Vasodilators


10. An uncommon cardiac manifestation of digitalis toxicity is:
1. premature ventricular contractions
2. second and third degree heart block
3. atrio-ventricular junctional escape beats
4. atrial fibrillation with rapid ventricular response
5. All are correct

Item Number: 4751 correct answer: 2 category: Inotropes/Vasodilators


11. The most widely used digitalis glycosides in the United States are digoxin and
digitoxin. The half-life (t1/2) of digitoxin is approximately:
1. 1 day
2. 7 days
3. 12 days
4. dependent upon the route of administration
5. dependent upon renal function

Item Number: 119 correct answer: 1 category: Inotropes/Vasodilators


12. The maintenance dose of digoxin is primarily dependent upon:
1. renal function
2. sex, hepatic function and protein level
3. pulmonary function
4. the loading dose and the type of diuretic being used
5. level of serum potassium
Item Number: 11408 correct answer: 4 category: Inotropes/Vasodilators
13. Thiocyanate toxicity is a potential adverse effect associated with:
A. amrinone
B. nitroglycerin
C. milrinone
D. nitroprusside
1. A,B,C
2. A,C
3. B,D
4. D only
5. All are correct

Item Number: 2722 correct answer: 4 category: Inotropes/Vasodilators


14. A 42-year-old white male with rheumatic mitral insufficiency is seen with
symptoms and findings of initial left ventricular failure. You give him
digitalis and recall that all of the following are pharmacologic effects of
digitalis EXCEPT:
1. decreases conduction velocity in the A-V node
2. increases the effects of normal vagal activity of the heart
3. shortens the effective refractory period of the Purkinje fibers
4. weakens myocardial contractility
5. prolongs the functional refractory period of the A-V node

Item Number: 1193 correct answer: 3 category: Inotropes/Vasodilators


15. Orthostatic hypotension is most likely to occur following the use of:
1. phenylephrine
2. reserpine
3. amyl nitrite
4. metaraminol
5. tetrahydrozoline

Item Number: 4600 correct answer: 2 category: Inotropes/Vasodilators


16. The chemical configuration of a molecule may determine its degree of
absorption, its distribution, and its route of metabolism. Structurally, the
digitalis glycosides resemble:
1. catecholamines
2. steroids
3. salicylates
4. nitrofurantoin
5. phenothiazines

Item Number: 13697 correct answer: 3 category: Inotropes/Vasodilators


17. All of the following are actions of nitrates in angina pectoris EXCEPT:
1. reflex increase in heart rate
2. ventricular size decrease
3. ejection fraction increased
4. peripheral venous pooling–decrease in preload
5. systolic blood pressure decrease

Item Number: 11398 correct answer: 4 category: Inotropes/Vasodilators


18. The sympathomimetic which may promote diuresis by a direct effect on the
kidney is:
1. isoproterenol
2. dobutamine
3. norepinephrine
4. dopamine
5. epinephrine

Item Number: 13704 correct answer: 4 category: Inotropes/Vasodilators


19. Nitroglycerin administered sublingually reduces venous tone, causing pooling
of blood in peripheral veins and lowers peripheral arterial resistance,
resulting in a decline in blood pressure. Both end-diastolic and end-systolic
dimensions of the left ventricle are reduced; therefore myocardial oxygen
needs are decreased primarily by a reduction of:
1. blood pressure
2. heart rate
3. coronary vascular resistance
4. intramyocardial tension
5. sulfhydryl bonds

Item Number: 8090 correct answer: 3 category: Inotropes/Vasodilators


20. Digitalis must be used with caution in patients with acute myocardial
infarction. Which of the following is true of digitalis use?
A. in normal hearts, it increases contractility and myocardial O2
consumption
B. it is recommended in uncomplicated myocardial infarction
C. in failing hearts, it decreases or leaves unchanged myocardial O2
consumption
D. in acute myocardial infarction, patients are less sensitive to the
development of arrhythmias due to digitalis toxicity
1. A only
2. A,B,C
3. A,C
4. B,D
5. D only

Item Number: 1160 correct answer: 3 category: Inotropes/Vasodilators


21. Digitalis may slow the heart rate by:
1. blocking the activity of the carotid and aortic arch baroreceptors
2. exerting an atropine-like effect on the A-V node
3. stimulation of vagal center in the medulla to increase efferent firing
4. increasing the rate of conduction through the A-V node
5. None of the above

Item Number: 11412 correct answer: 4 category: Inotropes/Vasodilators


22. Compared to digitoxin, digoxin is or has:
1. more highly protein-bound
2. more extensive enterohepatic circulation
3. prolonged elimination half-life
4. less completely absorbed orally
5. more extensively hepatically cleared
Item Number: 11390 correct answer: 4 category: Inotropes/Vasodilators
23. The beneficial effects of digitalis are derived from its effect on myocardial
contractility and on myocardial conduction and excitability. In therapeutic
doses, digitalis:
1. slows the atrial rate in atrial fibrillation
2. enhances atrioventricular conduction
3. slows the atrial rate in sinus tachycardias
4. slows the ventricular rate in atrial fibrillation
5. decreases ventricular automaticity

Item Number: 4696 correct answer: 3 category: Inotropes/Vasodilators


24. Digitalis-induced emesis is:
1. only seen when digoxin is given orally
2. of no true clinical importance
3. due to stimulation of the chemoreceptor trigger zone
4. related to Na+ K+ ATPase inhibition within the GI tract
5. commonly seen with rapid parenteral administration

Item Number: 4593 correct answer: 4 category: Inotropes/Vasodilators


25. Digitalis has a beneficial effect on the failing myocardium via its action to
increase the availability of calcium to the contractile machinery. As with
many drugs, digitalis has also been overused. The correct statement
concerning the use of digitalis in the ischemic and non-ischemic myocardium
is:
1. there is a net increase in oxygen consumption in the non-failing
myocardium
2. there is a net increase in oxygen consumption in the failing dilated
myocardium
3. digitalis has proven value in prophylaxis for diastolic dysfunction
4. digitalis causes vasoconstriction in the normal heart secondary to
enhanced sympathetic outflow and blocking the Na+-K+ ATPase in smooth
muscle
5. digitalis causes an increase in heart rate in the normal heart

Item Number: 11423 correct answer: 1 category: Antiarrhythmics


1. Properties of the class IA antiarrhythmics include
A. reduce automaticity of his-Purkinje fibers
B. decrease the rate of rise and amplitude of phase 0 depolarization
C. prolong P-R and Q-T intervals
D. facilitate inward sodium conductance during rapid depolarization
1. A,B,C
2. A,C
3. B,D
4. D only
5. All are correct

Item Number: 11432 correct answer: 4 category: Antiarrhythmics


2. Which of the following antiarrhythmics may worsen angina symptoms by
increasing myocardial oxygen requirements?
A. lidocaine
B. verapamil
C. propranolol
D. disopyramide
1. A,B,C
2. A,C
3. B,D
4. D only .
5. All are correct

Item Number: 11433 correct answer: 4 category: Antiarrhythmics


3. Cinchonism is a dose-related adverse reaction associated with which of the
following antiarrhythmics?
A. tocainide
B. procainamide
C. disopyramide
D. quinidine
1. A,B,C
2. A,C
3. B,D
4. D only
5. All are correct

Item Number: 11447 correct answer: 4 category: Antiarrhythmics


4. Addition of propranolol to the drug regimen of a patient receiving lidocaine
for premature ventricular contractions after a myocardial infarction results
in:
A. increased likelihood of breakthrough PVCs
B. displacement of protein-bound lidocaine
C. reduced renal clearance of lidocaine
D. increased likelihood of lidocaine toxicity
1. A,B,C
2. A,C
3. B,D
4. D only
5. All are correct
Item Number: 11448 correct answer: 1 category: Antiarrhythmics
5. Electrophysiologic properties possessed by lidocaine include:
A. reduction of effective refractory period in normal his-Purkinje
fibers
B. prolonged effective refractory period in ischemic his-Purkinje
fibers
C. shortened Q-T interval
D. prolonged P-R interval
1. A,B,C
2. A,C
3. B,D
4. D only
5. All are correct

Item Number: 11464 correct answer: 5 category: Antiarrhythmics


6. Which of the following are classified as IB antiarrhythmics?
A. lidocaine
B. phenytoin
C. tocainide
D. mexiletine
1. A,B,C
2. A,C
3. B,D
4. D only
5. All are correct

Item Number: 11466 correct answer: 1 category: Antiarrhythmics


7. In treating supraventricular arrhythmias, which of the following reduces
ventricular response primarily by increasing the effective refractory period
of the A-V node?
A. propranolol
B. digoxin
C. verapamil
D. tocainide
1. A,B,C
2. A,C
3. B,D
4. D only
5. All are correct
Item Number: 11474 correct answer: 1 category: Antiarrhythmics
8. Potential side effects associated with amiodarone therapy include:
A. pneumonitis
B. pseudocyanosis
C. photosensitivity
D. parotiditis
1. A,B,C
2. A,C
3. B,D
4. D only
5. All are correct

Item Number: 11475 correct answer: 3 category: Antiarrhythmics


9. Class IC antiarrhythmics possess which of the following electrophysiologic
properties?
A. markedly prolong repolarization
B. markedly depress phase 0 depolarization
C. inhibit calcium entry during plateau portion of action potential
D. inhibit sodium entry during phase 0 depolarization
1. A,B,C
2. A,C
3. B,D
4. D only
5. All are correct

Item Number: 11476 correct answer: 1 category: Antiarrhythmics


10. Adverse effects associated with disopyramide therapy include:
A. urinary retention
B. constipation
C. blurred vision
D. lupus syndrome
1. A,B,C
2. A,C
3. B,D
4. D only
5. All are correct

Item Number: 289 correct answer: 5 category: Antiarrhythmics


11. Antiarrhythmic drugs can be separated into four groups, types I to IV. All of
the following are type I antiarrhythmic drugs EXCEPT:
1. quinidine
2. lidocaine
3. procainamide
4. phenytoin
5. propranolol

Item Number: 291 correct answer: 5 category: Antiarrhythmics


12. Individual antiarrhythmic agents can alter the surface electrocardiogram.
Characteristics of quinidine include:
1. decreased PR interval
2. increased QRS duration
3. increased QT interval
4. All of the above
5. 2,3

Item Number: 292 correct answer: 5 category: Antiarrhythmics


13. Quinidine toxicity includes all of the following EXCEPT:
1. thrombocytopenia
2. diarrhea
3. cinchonism
4. ventricular fibrillation
5. hypertension

Item Number: 293 correct answer: 1 category: Antiarrhythmics


14. Of the following antiarrhythmic agents, the one which most closely resembles
procainamide with respect to electrophysiologic effects and mechanism of
action is:
1. quinidine
2. lidocaine
3. phenytoin
4. propranolol
5. bretylium

Item Number: 1360 correct answer: 5 category: Antiarrhythmics


15. Bronchiolar constriction is a rare but potentially dangerous side effect of:
1. quinidine
2. lidocaine
3. procainamide
4. phenytoin
5. propranolol

Item Number: 1578 correct answer: 4 category: Antiarrhythmics


16. The effect of different antiarrhythmic agents is best understood by knowing
their predominant actions on cardiac fibers. All of the following general
statements are true EXCEPT:
1. quinidine slows the rate of depolarization of cardiac action potentials
and increases the refractory period
2. lidocaine and phenytoin are class 1 B antiarrhythmic
3. beta-receptor blocking drugs act by reducing the slope of the pacemaker
potential
4. procainamide acts as a specific calcium antagonist
5. bretylium prolongs the action potential and so prolongs the absolute
refractory period

Item Number: 9518 correct answer: 4 category: Antiarrhythmics


17. In the pharmacologic treatment of cardiovascular disorders, quinidine has all
of the following effects EXCEPT:
1. prolongation of the effective refractory period
2. slows conduction velocity
3. reduces spontaneous frequency of discharge of ectopic pacemaker
4. has a positive chronotropic effect
5. produces peripheral vasodilation in large doses

Item Number: 9893 correct answer: 3 category: Antiarrhythmics


18. Which of the following medications is contraindicated in a patient with
untreated complete heart block?
1. atropine
2. prednisone
3. quinidine
4. isoproterenol
5. hydrochlorothiazide

Item Number: 11386 correct answer: 1 category: Antiarrhythmics


19. The therapeutic rationale for the use of quinidine or procainamide includes
their ability to:
1. depress ectopic myocardial automaticity
2. enhance myocardial membrane responsiveness
3. decrease the effective refractory period
4. 1,2
5. 2,3

Item Number: 12788 correct answer: 3 category: Antiarrhythmics


20. All of the following are common side effects of quinidine administration
EXCEPT:
1. diarrhea
2. nausea and vomiting
3. dry mouth
4. tinnitus
5. headache

Item Number: 14457 correct answer: 4 category: Antiarrhythmics


21. 24 hours after an acute myocardial infarction, a 46-year-old male is being
treated with a continuous intravenous drip of an antiarrhythmic drug to
suppress frequent multifocal premature ventricular contractions. He develops
generalized seizure activity. The seizure activity can be most readily
explained by:
1. ventricular tachycardia
2. systemic embolization
3. systemic hypotension
4. lidocaine toxicity
5. ventricular asystole

Item Number: correct answer: 1 category: Antiarrhythmics


22. TRUE statements concerning Vaughan-Williams’s Class III antiarrhythmic
drugs include all EXCEPT:
1. encainide, flecainide, and propafenone are in this class
2. bretylium, amiodarone, and sotalol are in this class
3. possess diverse pharmacologic effects but share the capacity to prolong
action potential duration and refractoriness in Purkinje fibers
4. block outflow of potassium during cell repolarization
5. do not alter phase 0 depolarization or resting membrane potential

Questions 23 – 29
Choose the correct answer for the following questions from the list below.
Each answer can be used once, more than once, or not at all.

1. bretylium
2. amiodarone
3. adenosine
4. flecainide
5. lidocaine
Item Number: correct answer: 5 category: Antiarrhythmics
23. In ACLS algorithm, first antiarrhythmic to be administered in the treatment of
ventricular fibrillation refractory to defibrillation and epinephrine.

Item Number: correct answer: 3 category: Antiarrhythmics


24. Drug of choice for paroxysmal supraventricular tachycardia unresponsive to
vagal maneuvers.

Item Number: correct answer: 1 category: Antiarrhythmics


25. Produces transient increase in blood pressure followed by hypotension due to
an initial release of norepinephrine from neurons followed by blockade of
norepinephrine re-uptake.

Item Number: correct answer: 2 category: Antiarrhythmics


26. Contains iodine in molecular structure, is similar to thyroxine, and use is
associated with hyper- or hypothyroidism in 2% – 5% of patients.

Item Number: correct answer: 2 category: Antiarrhythmics


27. Pharmacokinetic properties characterized by very high protein-binding (99.9%),
very large volume of distribution (66 L/kg), and average elimination half-life
of 25 days.
Item Number: correct answer: 4 category: Antiarrhythmics
28. A class IC antiarrhythmic, characterized by high affinity for and slow
dissociation from fast sodium channels

Item Number: correct answer: 5 category: Antiarrhythmics


29. A class IB antiarrhythmic, characterized by rapid association – dissociation
from sodium channels

Item Number: correct answer: 4 category: Antiarrhythmics


30. Correct statements concerning propafenone include all EXCEPT:
1. class IC antiarrhythmic
2. slows actions potential conduction by blocking sodium channels
3. possesses weak beta blocking activity
4. class III antiarrhythmic
5. increases duration of QRS complex

Item Number: correct answer: 1 category: Antiarrhythmics


31. Correct statements concerning sotalol include all EXCEPT:
1. prolongs action potential duration and effective refractory period by
blocking sodium channels.
2. prolongs cell repolarization by blocking potassium channels.
3. produces bradycardia
4. prolongs Q-T interval
5. increases effective refractory period

Questions 32 – 35
Choose the correct answer for the following questions from the list below.
Each answer can be used once, more than once, or not at all.
1. propranolol
2. acebutolol
3. sotalol
4. esmolol
5. butorphanol

Item Number: correct answer: 4 category: Antiarrhythmics


32. Cardioselective beta blocker with a short elimination half-life administered
intravenously only.

Item Number: correct answer: 3 category: Antiarrhythmics


33. Nonselective beta blocker which slows heart rate and also prolongs action
potential duration by blocking transmembrane potassium currents.
Item Number: correct answer: 2 category: Antiarrhythmics
34. Cardioselective beta blocker possessing intrinsic sympathetic activity and
given by oral administration only.

Item Number: correct answer: 1 category: Antiarrhythmics


35. Nonselective beta blocker possessing quinidine-like membrane stabilizing
effects.

Item Number: correct answer: 5 category: Antiarrhythmics


36. FALSE statement concerning use of calcium channel blockers as antiarrhythmics:
1. slows inward calcium current thereby decreasing the rate of spontaneous
phase 4 depolarization in Purkinje fibers
2. slows conduction velocity through the atrio-ventricular node and
increases functional refractory period
3. useful for slowing ventricular rate in atrial fibrillation
4. hypotension may be a limiting side effect
5. verapamil, diltiazem, and nifedipine all exert equally effective
antiarrhythmic actions
Item Number: correct answer: 3 category: Antiarrhythmics
37. CORRECT statement concerning the antiarrhythmic drug adenosine:
1. undergoes extensive hepatic metabolism
2. produces coronary vasoconstriction
3. causes frequent but transient post-conversion arrhythmias
4. increases sinus node rate
5. increases atrio-ventricular conduction

Item Number: correct answer: 1 category: Antiarrhythmics


38. True statement concerning adenosine include all EXCEPT:
1. administered sublingually
2. negative chronotrope
3. negative dromotrope
4. negative inotrope
5. may precipitate bronchospasm

Item Number: correct answer: 5 category: Antiarrhythmics


39. FALSE statement concerning adenosine:
1. half-life of 1 – 10 seconds
2. higher doses necessary in patients taking theophylline
3. chest pain and shortness of breath are common but short-lived side
effects
4. indicated for treating paroxysmal supraventricular tachycardia
5. less effective than verapamil for treating paroxysmal supraventricular
tachycardia

Item Number: correct answer: 2 category: Antiarrhythmics


40. Correct statements about epinephrine administration for ventricular
fibrillation include all EXCEPT:
1. 1 mg IV push every 3 – 5 minutes
2. 0.1 mg IV push every 3 – 5 minutes
3. 0.1 mg/kg IV push every 3 – 5 minutes
4. 1 mg IV push followed by 3 mg IV push followed by 5 mg IV push 3 minutes
apart
5. 2.5 mg diluted to a total volume of 10 ml in saline solution via an
endotracheal tube when no IV access is available

Buprenorphine is a?
A. Partial mu agonist
B. Full mu agonist
C. Partial kappa agonist
D. Full kappa agonist

Which of the following is an integrase inhibitor currently in use for the treatment of HIV infection?
A. Indinavir
B. Elvitegravir
C. Saquinavir
D. Raltegravir

Item Number: 10968 correct answer: 2 category: Anticoagulants


1. A patient diagnosed with a deep venous thrombosis is begun on a heparin
infusion of 1000 U/HR after a 5000 U Bolus. The earliest you can draw blood
for an aPTT reflecting steady-state heparin is:
1. 4 hours
2. 7.5 hours
3. 12 hours
4. 24 hours
5. None of the above

Item Number: 10973 correct answer: 2 category: Anticoagulants


2. The direct-acting plasminogen activator is:
1. streptokinase
2. urokinase
3. epsilon-aminocaproic acid
4. anistreplase
5. heparin

Item Number: 10980 correct answer: 4 category: Anticoagulants


3. The antithrombotic drug which inhibits cyclooxygenase enzymes is:
1. prednisone
2. dipyridamole
3. tranexamic acid
4. aspirin
5. ticlopidine

Item Number: 10983 correct answer: 3 category: Anticoagulants


4. Correct statements about the action of warfarin include:
A. warfarin is structurally similar to vitamin K and competitively
inhibits liver synthesis of clotting factors
B. warfarin inhibits vitamin K epoxide reductase
C. the onset of warfarin action may be prolonged due to the long
half-life of factor VII
D. factors II, VII, IX, and X are not activated (carboxylated) when
warfarin is administered in therapeutic doses
1. A,B,C
2. A,C
3. B,D
4. D only
5. All of the above

Item Number: 10988 correct answer: 3 category: Anticoagulants


5. Drugs which interact with warfarin to increase its anticoagulant effect
include:
A. ranitidine
B. trimethoprim/sulfamethoxazole
C. rifampin
D. nonsteroidal anti-inflammatory drugs
1. A,B,C
2. A,C
3. B,D
4. D only
5. All of the above

Item Number: 11018 correct answer: 1 category: Anticoagulants


6. The mechanism of action of mini-dose heparin is correctly described by:
A. factor X is more sensitive to heparin than are other serine
protease factors
B. mini-dose heparin is useful prophylactically rather than during
active clot formation
C. by inhibiting factor X activation, a relative block of the
clotting cascade develops, with less thrombin formed
D. the major action is to directly inhibit factor II, which is very
sensitive to heparin
1. A,B,C
2. A,C
3. B,D
4. D only
5. All of the above

Item Number: 11041 correct answer: 2 category: Anticoagulants


7. The correct statement about fibrinolytic agents is:
1. streptokinase acts directly to activate plasminogen
2. urokinase acts directly to activate plasminogen
3. tissue-type specific plasminogen activator is more fibrin specific than
streptokinase
4. side effects of t-PA include skin rash, allergic reactions, and
development of antibodies
5. have been shown to increase mortality when given in the setting of an
acute myocardial infarction

Anda mungkin juga menyukai